Skip to content

Instagram Posts

Featured Post type Major topic Minor topic Diagnosis Learning points Significant images Series Full post text Instagram link
Clinical Case FAST Normal Fast Mirror artifact You have a 23 year old male that was involved motor vehicle collision arrive to your ED via ambulance. He is tachycardic to 115 with a normal blood pressure. He is complaining of significant right sided pain on his chest. You are able to obtain the above view of the right lung base. The trauma attending asks if the patient has a hemothorax, what do you tell them? -- Answer Below -- Answer: Not that you can see, this is a normal appearing right lung base Ultrasound is very good at detecting pleural effusions. While around 150cc of fluid is required to identify an effusion on upright CXR, in the hands of an experienced sonographer, a pleural effusion as small as 5cc can be identified. If you took a look at this image and thought it was positive for a pleural effusion you are likely not alone. What you are seeing here above the diaphragm is something called mirror artifact. The diaphragm is very good at reflecting the sound waves from the ultrasound probe and in normal air filled lung tissue there is nothing for the ultrasound waves to bounce off of. Therefore if the lung is normal beyond the diaphragm, the ultrasound will interpret liver that is in front of the diaphragm as also being behind it. The key to deciding if you are looking at mirror artifact vs a true effusion is to find the spine. As you can see in the far field of this image the spine terminates right at the diaphragm, if there was fluid in the posterior recess sound waves from the ultrasound would travel through the fluid and give you a view of the spine above the diaphragm. https://www.instagram.com/p/CJHgAHvHKBz/
Clinical Case Gallbladder Cholelithiasis cholelithiasis, findings of cholecystitis You are seeing a 43 year old female in the ED. She is having right upper quadrant pain. The point of care ultrasound (POCUS) is shown above. No measurements are abnormal. What is your diagnosis and how are you going to manage the patient? -- Answer Below -- Diagnosis: Cholelithiasis Management: Discharge after pain control with surgery follow up This ultrasound shows a normal appearing gallbladder (GB) with a large stone. Even though this stone is big, without findings of cholecystitis or a dilated CBD, if you can control the patients pain then they can be discharged with outpatient surgery follow up. So what are the ultrasound findings of cholecystitis? There are 5: 1. Thickened anterior GB wall >3mm 2. Pericholecystic fluid 3. Sonographic Murphy’s sign 4. Hydropic GB, which is a GB measuring >10cm long x 5cm wide 5. Non-mobile stone lodged in the GB neck https://www.instagram.com/p/CJM8ObQHMn_/
Clinical Case Bowel Diverticulitis A 60 year old male walks into your urgent care clinic. He says that he has had mild pain in his LLQ for the past 14 hours. He notes that he had a colonoscopy done that showed diverticula but he has never had diverticulitis. You perform ultrasound shown above. How are you going to manage this patient? Does he need to be sent to the emergency room for a CT scan? -- Answer Below -- Diagnosis: Diverticulitis Disposition: Uncomplicated diverticulitis should be managed outpatient with antibiotics and GI follow up. There is currently some debate about whether or not uncomplicated diverticulitis even needs antibiotics. So how do you find diverticulitis on ultrasound? Well according to Cohen et al 2020 it is relatively easy to learn and has a high sensitivity and specificity. In her study Dr. Cohen taught fellows and attendings via a 40 minute didactic section and then they completed 5 precepted scans. They found that ultrasound had a 97% specificity and a 92% sensitivity for detecting diverticulitis compared to CT scan. They also noted that in 100% of complicated diverticulitis ultrasound identified findings consistent diverticulitis. In this ultrasound image you see the dark wall of the bowel and small bright white foci of air. Bowel lumen is usually air and fluid filled and can look like mixed dirty shadowing or just a strip of bright white. The smaller white foci here likely represent intramural diverticula. You can also see a segment of hyperechoic fat behind the bowel that is consistent with localized inflammation. The other findings of diverticulitis on ultrasound are: -A hypoechoic peridiverticular inflammatory reaction -Mural and peridiverticular abscess formation with or without gas bubbles -Bowel wall thickening (segmental mural thickening greater than 4 mm) at the point of maximal tenderness -Presence of diverticula in the surrounding segments https://www.instagram.com/p/CJM9-xEHZ--/
Clinical Case Cardiac Normal diastolic function Diastolic assessment, E wave, A wave You have a 56 year old female who presents with acute shortness of breath. You notice that her LV function looks normal but she has pulmonary edema on lung ultrasound. You move back to the apical 4 chamber view and obtain this measurement. What are we measuring here and is this pattern normal or abnormal? How would you manage the patient and what is the diagnosis? -- Answer Below -- Diagnosis: Normal diastolic function Management: Continue work up for other causes of pulmonary edema Here we are looking at the mitral valve inflow pattern and evaluating the patient's diastolic function. As you can see from the right upper part of the image we are measuring the E wave and the A wave velocities. You can do this by placing a pulsed wave doppler gate at the opening of the mitral valve leaflets in the apical 4 chamber view. The E wave represents passive filling of the left ventricle (LV) due to a negative force created by ventricular relaxation. The A wave represents atrial contraction, which normally does not contribute much to filling of the ventricle. Over time, usually from long standing hypertension, the left ventricle can become stiff and begins to have impaired relaxation. Without proper relaxation there is not enough negative pressure generated within the LV to cause a significant influx of blood during the passive phase. This will change the pattern so that the A wave becomes bigger than the E wave. A pattern of E>A would represent the early stages of diastolic heart failure. There are more components to complete diastolic analysis that we will discuss in later posts. Since our patient here has findings concerning for pulmonary edema in the setting of normal LV as well as diastolic heart function, we should assume that our lung findings are due to something other than heart failure and continue our evaluation of the patient. https://www.instagram.com/p/CJM_bI-Hy19/
Clinical Case Gallbladder Cholelithiasis findings of cholecystitis You have a 60 year old male with epigastric pain for the past 10 hours. You perform the ultrasound shown above and find that the common bile duct (CBD) measures 0.54 cm and anterior gallbladder (GB) wall measurement is normal. Is this patient’s gallbladder that cause of his pain? What should be your next step in management? -- Answer Below -- Diagnosis: Normal Gallbladder Ultrasound Disposition: Since the GB US is normal you should continue to work up the cause of the patients pain. Don’t forget that a normal CBD measurement is somewhere around 7mm. The general rule of thumb is that CBD should be 1mm for every decade of life after 40 years old, so a 40 year old should have a CBD that measures 4mm or less, a 60 year old should have one that is 6mm or less. That being said, even in younger people a CBD measurement <7mm can be normal and in those cases clinical context is important. After a cholecystectomy CBD can measure up to 10mm and be normal. Do you remember the ultrasound findings of cholecystitis? There are 5: 1. Thickened anterior GB wall >3mm 2. Pericholecystic fluid 3. Sonographic Murphy’s sign 4. Hydropic GB, which is a GB measuring >10 cm long x 5cm wide 5. Non-mobile stone lodged in the GB neck https://www.instagram.com/p/CJM_6aPng4q/
Clinical Case Obstetrics (OB) Pregnancy of unknown location features of definitive intrauterine pregnancy gestational sac A 26 year old female presents to your ED, she believes that she is 5 weeks pregnant and is presenting for 4 hours of lower abdominal pain. Her blood pressure is 128/90 and her heart rate is 86. She appears mildly uncomfortable and a Beta HCG returns at 2000. Point of care ultrasound (POCUS) is done and the image is shown above. Based on this image what is your diagnosis? What should be the next step in managing this patient in the ED. -- Answer Below -- Diagnosis: Pregnancy of unknown location Management: This patient should undergo a transvaginal ultrasound to better visualize the gestational sac as well as evaluate for ectopic pregnancy. The first definitive sign of intrauterine pregnancy on ultrasound is a yolk sac. In the above images you can see a small gestational sac within the uterus without a yolk sac. The issue here is that an ectopic pregnancy may have an associated pseudo gestational sac that may be falsely reassuring. The next step in managing this patient is to have them undergo a transvaginal ultrasound (TVUS) that will allow for better visualization of the gestational sac and may even visualize an internal yolk sac. If there is still no yolk sac seen on TVUS and there is no evidence of an ectopic pregnancy then she should follow up with an OBGYN in 48 hours to have a repeat HCG and ultrasound performed. During the first trimester the HCG should approximately double every 48 hours in normal pregnancy. There is no establish BHCG cut off where either transvaginal or transabdominal ultrasound should always definitively visualize a yolk sac, therefore no matter what level the BHCG if there are no clinical or ultrasound findings concerning for ectopic, the patient can be discharged to follow up with OBGYN. https://www.instagram.com/p/CJNAwOonfC3/
Clinical Case Lung Hypoxia air bronchograms static air bronchograms, atelectasis ou have an 86 year old female arrive to your ED with compressions ongoing. EMS reports they were called to her home where she collapsed and was found pulseless by a family member who began CPR. They report an initial rhythm of PEA, 2 doses of epinephrine were given en route. You perform an ultrasound and see the following image, what do you think is the etiology of her PEA arrest? -- Answer Below -- Likely Etiology: Hypoxia This image shows extensive abnormalities of the lung tissue. On the right side of the image you can see the diaphragm with an overlying effusion. The lung appears densely consolidated with multiple static air bronchograms. These findings appear consistent with significant lung disease on the right side, while static air bronchograms are usually associated with atelectasis, the extent of them throughout this entire lung likely represents a severe pneumonia. https://www.instagram.com/p/CJNBfsRn6fq/
Clinical Case Cardiac Congestive heart failure (CHF) ejection fraction (EF) A 54 year old male arrives with acute onset shortness of breath (SOB). He has a history of asthma but says that he has no other medical issues. You are concerned that his heart could be contributing to his SOB and obtain the above echo images. Would you classify this patients ejection fraction (EF) as normal, decreased, or severely decreased? -- Answer Below -- Answer: Severely decreased In general ejection fraction is broken up into several categories: -Hyperdynamic = >70% -Normal = 50-70% -Mildly reduced = 40-49% -Moderately reduced = 30-39% -Severely reduced = <30% In the ED we usually just think about >50%, 30-50, or less than 50% as normal, moderate, or severe. While there are multiple ways to estimate and quantify a patients ejection fraction visual estimation is by far the most widely used in the ED. Comprehensive echos will use something called Simpson’s method of disks to get a more accurate ejection fraction, however in the acute emergency exact EF is less important than understanding where the patient sits on the spectrum of dysfunction. There are several quick methods to estimate EF such as EPSS and fractional shortening, however these have never been shown to add anything to the visual estimation of an experienced ED clinician. https://www.instagram.com/p/CJNC0pdHQ2z/
Clinical Case FAST Positive FAST positive right upper quadrant fast A 24 year old female arrives with abdominal pain and vaginal bleeding. She states that she thinks she missed her last few periods. Her heart rate is 112 and her blood pressure is 80/60. You are concerned for an immediately life-threatening diagnosis, so you perform a point of care ultrasound (POCUS) and see the above image. What is your diagnosis and what is the next steps in management? -- Answer Below -- Diagnosis: Ruptured ectopic pregnancy Management: This patient needs urgent OBGYN evaluation for definitive management in the operating room, so your first step should be to call for help. Establish 2 large bore IVs or other good intravenous access. Make sure to order a type and screen along with several units of unmatched blood. This patient is in hemorrhagic shock so if she continues to be hypotensive despite 3 units of unmatched blood then a massive transfusion protocol (MTP) should be activated. It would not be unreasonable to activate an MTP immediately depending on your institutions policies. The above images shows a positive right upper quadrant (RUQ) FAST window. The FAST exam stands for focused assessment with sonography in trauma. In a supine adult patient the RUQ view is the most sensitive for free fluid in trauma and should be where you look first for free fluid. To call the RUQ truly negative you need to visualize the lower pole of the liver and or the kidney, whichever is more inferior. On the LUQ you need to see the potential space between the spleen and the diaphragm to truly call that quadrant negative. Although this case was not a trauma, a positive fast scan showing free intraperitoneal fluid in a previously healthy patient is assumed to be blood until proven otherwise. https://www.instagram.com/p/CJNDxc9HJB6/
Clinical Case Cardiac Tachycardia hyperdynamic ejection fraction (EF) hyperdynamic EF You are working overnight in a small rural hospital. A nurse calls you to an admitted patients bedside because the patient is feeling short of breath and appears diaphoretic. You have an ultrasound available and take a look at the patient’s heart before they get hooked up to any other monitoring devices. What do you think is going on with this patient and what is your next step in management? -- Answer Below -- Answer: Significant tachycardia limiting ventricular filling Management: Since this patient is still not hooked up to any monitoring devices the next few steps should be evaluating the rhythm on the EKG and the patients blood pressure. In arrhythmia management the patient’s mental status and blood pressure are major factors in determining acute management. Additionally an EKG showing sinus tachycardia should lead you to think that there is a secondary cause. Ejection fraction isn't everything when it comes to echocardiography. This image shows a very high ejection fraction, however, due to very poor LV pre-load, the cardiac output is very poor. If the EKG shows a tachyarrhythmia other than sinus tachycardia the answer is likely to slow the patient’s heart rate down to increase ventricular filling therefore increasing cardiac output. Remember cardiac output = heart rate x stroke volume. Stroke volume and ejection fraction are related but not the same. You can use something called VTI on ultrasound to get a better idea of a patients stroke volume and that is something we will discuss in a later post. https://www.instagram.com/p/CJPhq1wHn-M/
Clinical Case Cardiac transesophageal (TEE) Normal TEE Mid-esophageal four chamber view (ME4) Mid-esophageal four chamber view (ME4) A 34 year old female was found unconscious by her friends who called 911. Upon arrival EMS states that the patient was completely unresponsive so she was intubated for airway protection. In your ED the patient has a strong pulse but continues to be unresponsive to external stimuli. You obtain the above ultrasound image. What view is this view called and what is your interpretation of the patients cardiac function? -- Answer Below -- Answer: This is a mid esophageal 4 chamber view using the transesophageal (TEE) probe. The patient’s cardiac function appears normal. TEE, while it is a more invasive method, provides a much better view of the heart allowing for greater diagnostic accuracy for a variety of conditions. TEE can be used during cardiac arrest to continuously monitor the quality of compressions as well as watch for return of spontaneous circulation. https://www.instagram.com/p/CJPiJrznW0r/
Clinical Case Testicular Normal flow Normal testicular flow 24 year old male presents with acute onset severe left groin pain. He says that he has never had pain like this before. What can you see in this image and do you notice anything abnormal? -- Answer Below -- Answer: This is the testicular “buddy view.” You can see both the left and right testicles as well as the anterior scrotal skin. The color flow appears fairly symmetrical in both testicles and is reassuring when considering torsion as a potential diagnosis. That being said, there may be flow present in up to 25% of cases with surgically confirmed torsion. Ultrasound is still an excellent tool. However, if you are still concerned it could be torsion after a normal ultrasound, you should still call urology. The complete ultrasound exam for torsion includes several other elements. Each testicle should be evaluated individually for each of the following components: Arterial and venous pulsed wave doppler patterns Calculation of the resistive index, RI >0.75 may indicate early torsion 4-6 hours after torsion the affected testicle will have increased echogenicity compared to the other side Evaluate for spiral twisting of the spermatic cord https://www.instagram.com/p/CJPirpsns-Q/
Clinical Case Lung Congestive heart failure (CHF) Findings of congestive heart failure (CHF) pleural effusion, pulmonary edema, B lines You are on a busy ER shift and a 59 year old male is brought in by ambulance for acute onset shortness of breath (SOB). Patient notes that he has been treated for pneumonia for the past 2 weeks as an outpatient and has been bedridden. His vital signs include heart rate of 120, blood pressure of 185/110, pulse ox of 86% on room air. Patient appears to be in acute respiratory distress. Being the department point of care ultrasound (POCUS) champion your colleague asks you to come take a look at the patient because they are worried he might have a pulmonary embolism (PE). You perform a POCUS exam and obtain the above images. Does this patient have any signs of a PE on echo? Based on these images what is your current diagnosis and how should this change management? -- Answer Below – Diagnosis: Acute heart failure with pulmonary edema Management: The management of acute heart failure varies depending on the exact etiology, however in the ED the answer is generally to reduce the patients pre-load and optimize their position on the Frank Starling curve. Since this patient is hypertensive generally the first line treatment for pre-load reduction would be sublingual or IV nitrates. Another key step in the management of CHF is respiratory support with CPAP/BiPAP, which will provide higher concentrations of inspired O2 vs room air and also serve to increase intrathoracic pressure therefore further reducing pre-load. Ultrasound: The first few images show several lung abnormalities. In the first image you can see a moderate effusion above the diaphragm along with the tip of the lung coming in and out of view. The second and third images both show diffuse B-lines on the right as well as the left. The last image shows a large pleural effusion on the left side. Now, while all of these findings could be consistent with a severe pneumonia since B-lines are not specific to pulmonary edema......the next 2 images show a heart with severely reduced left ventricular function. The combination of these two findings suggest that your diagnosis is pulmonary edema from heart failure rather than SOB caused by a PE or worsening PNA. https://www.instagram.com/p/CJRGBNwnCVd/
Clinical Case Cardiac Congestive heart failure (CHF) acute global hypokinesis apical thrombus A 36 year old female presents to your emergency room with worsening lower extremity swelling and shortness of breath. She says that she has no past medical history. She notes 7 days ago she had some moderate right shoulder pain that has now gotten worse. You perform a point of care ultrasound (POCUS) and obtain the first three images. What do you notice in these images and given that the patient is a young female with no past medical history what is on your list of differentials? There are several images after the first three, wait to review them until you have read the answer section below. -- Answer Below – The first three images show a severely decreased ejection fraction and a small pericardial effusion. Normally in older adults heart failure is caused by long standing uncontrolled hypertension, valvular issues, or ischemic heart disease. In a young female with no prior medical history you should consider alternative causes. Common causes of heart failure among the young include adult congenital heart disease, cardiomyopathies, myocarditis, or alcohol/drug related myocardial lesions. Now moving onto the next few images in the series. This patient went for a CT scan of her abdomen due to abdominal pain. The CT identified a filling defect within the left ventricle concerning for a thrombus. The ED clinician returned to the bedside and obtained further apical images of the heart demonstrating thrombus within the left ventricle (LV) and that is what can be seen in the last few images. LV thrombus is rare. It is generally a complication seen after acute myocardial infarction (AMI). The rate of LV thrombus has decreased since the introduction of primary percutaneous coronary intervention (PCI). Prior to PCI, AMI would leave significant regional wall akinesia and dyskinesia resulting in blood stasis and therefore increased risk of thrombus formation. Wall akinesia and dyskinesia, of unknown cause in this case, are the likely cause of thrombus formation. https://www.instagram.com/p/CJg-v0bHcuv/
Clinical Case Renal UVJ kidney stone kidney stone passing rates, kidney stone treatment, twinkle sign UVJ stone, twinkle sign, hydronephrosis A 62 year old male presents to your emergency department with acute onset right flank pain. The patient says that he has a history of kidney stones and this feels similar. He admits to a few episodes of vomiting but hasn’t noticed any blood in his urine. You perform the above point of care ultrasound (POCUS). What is the diagnosis and how are you going to manage this patient in the ED? -- Answer Below – Diagnosis: Ureterovesical junction (UVJ) stone measuring <5mm Management: If you are able to control the patient’s pain in the ED and there are no signs of infection on the urinalysis, then the patient can be discharged home to pass the stone spontaneously. Stones 2-4 mm pass spontaneously at a rate of ~76% and 5-7 mm ~60%. Acute pain management in the ED should begin with intravenous (IV) Toradol with the addition of narcotics if Toradol alone is insufficient. The patient should be discharged with instructions to continue NSAIDs at home along with a prescription for several narcotic tablets for breakthrough pain episodes. The first ultrasound image shows the right kidney with mild hydronephrosis. The left kidney appears normal. In the last few images you can see the bladder with a hyperechoic structure just posterior to it on the left side of the image. Remember that the ultrasound probe indicator always faces to the patient’s right side when the probe is in the transverse orientation. So that means that this hyperechoic structure (stone) is on the patient’s right, the side where his pain is. You can see that in one of the images a color doppler box is placed around the area of interest. This is called “The Twinkle Sign” and has been described as a way to help increase urinary stone identification on POCUS. The sign is described as color artifact presenting as rapidly changing colors seen posterior to a reflector. https://www.instagram.com/p/CJhEp3KHWOQ/
Clinical Case Cardiac Normal VTI Volume assessment, cardiac output, velocity time integral (VTI), inferior vena cava (IVC) VTI calculation A 45 year old male arrives to your ED complaining of cough and shortness of breath. Vital signs show a temp of 39 C, Blood pressure 90/62, HR 115, SpO2 98% on room air. You are concerned that the patient is hypotensive from sepsis. Your attending asks if there is a way that you can use point of care ultrasound (POCUS) to evaluate if the patient is volume responsive. What is your response? -- Answer Below – While the inferior vena cava (IVC) measurement has been the “classic” way of evaluating fluid status in patients it is fraught with errors. 1. Most of the original literature regarding IVC and fluid responsiveness came from trauma literature. We have expanded this to most medical patients, which is not the same population. 2. There is no standardized location or way to measure the IVC in the literature. 3. There is no agreed on “extreme” or amount of respiratory variation that has showed adequate sensitivity or specificity for volume responsiveness. When it comes to being “volume responsive” what we are really talking about is increasing cardiac output. Cardiac output (CO) = Heart rate (HR) x Stroke volume (SV). SV = velocity time integral (VTI) x cross sectional area of the left ventricular outflow track (LVOT) The VTI is something that we can measure easily in an apical 5 chamber view. The first image above shows what that looks like. Once you have a good apical 4, just fan the ultrasound beam more anterior until you see the LVOT exiting the LV. Once you have that view place a pulsed wave doppler gate within the LVOT. The 2nd image shows the tracing of the LVOT VTI. You will use the machines calc package and trace the negative deflection that represents the blood flowing out of the LVOT. Now since SV=VTI x LVOT area, if we assume the LVOT area is somewhat constant in our single patient, we can simplify that equation to say SV=VTI. Now if our goal is to increase CO with fluids, then serial VTI measurements after either passive leg raise or a fluid bolus will tell us if our patient is fluid responsive with much greater accuracy than any other method. Don’t forget to take HR into account though! Future posts will cover more on the IVC and VTI pearls. https://www.instagram.com/p/CJoV_FFnhuM/
Clinical Case Cardiac Normal right ventricular (RV) function TAPSE, D-sign, RV to LV ratio TAPSE measurement A 35 year old female presents to the emergency room with shortness of breath (SOB), she has no past medical history. She states that she has associated pleuritic chest pain and is taking oral contraceptive pills that contain estrogen. Her vital signs are HR 104, BP 130/80, SpO2 100% on RA. You are concerned for a pulmonary embolism (PE). You perform the above point of care ultrasound (POCUS). Based on your findings above do you think that the patient has a PE? -- Answer Below – Do you think your patient has a PE? – No There are several indicators on POCUS of acute PE. “The D-sign” is described as the septal wall between the LV and RV appearing flat, this is because in acute PE, RV pressures rise and cause the septum to move towards the LV. As you can see here the septum appears normal (curved toward RV). “D-sign” has a positive likelihood ratio (LR) of 5.3 and a negative LR of 0.53. Next, LV to RV ratio. Look at the size of the LV:RV in the apical 4 chamber (AP4) view (image 2 and 3). As RV pressures increase with acute PE, RV size also increases. Here, the RV is smaller than the LV, thus the ratio is normal. A ratio of > 1:1 has a +LR of 2.6 and <1:1 a – LR of 0.73. Check out the post on how to interpret LRs, but let me tell you LV:RV ratio isn’t particularly helpful in diagnosing PE. The third image shows something called tricuspid annular plane systolic excursion (TAPSE). To get this measurement, get an AP4 view, place an M mode line over the free wall portion of the tricuspid anulus and measure the peak to trough. TAPSE shows how well the RV is contracting and gives you an idea of RV failure/strai. TAPSE <20mm has a sensitivity of 72% and specificity of 66% for PE. Note that TAPSE can be abnormal in other situations as well but in a young person without history of RV failure an abnormal TAPSE is concerning for PE. Here we can see the TAPSE is 25mm therefore considered normal. https://www.instagram.com/p/CJrJZTpn0Kh/
Clinical Case Ocular Retinal detachment Ocular ultrasound setup A 58 year old male presents to your emergency room complaining of “flashing lights” in his right eye for the past hour. The external exam of the eye appears normal. You are concerned that something else could be going on but aren’t comfortable doing a dilated fundoscopic exam, what other options do you have? Well….Point of care ultrasound (POCUS)! What diagnosis do you notice in the 2nd image? -- Answer Bellow – Diagnosis: Retinal detachment Ocular ultrasound is actually one of the easiest and most satisfying POCUS exams you can do. I like to place a tegederm over the patients closed eye and then put a generous amount of gel on top of the tegederm. Use the linear probe and a light amount of pressure (really you should be floating the probe on top of your gel). A mistake I notice a lot of novice scanners make is just keeping the probe oriented strictly transverse. The eye is round so just slowly rotate the probe until you can see the entire globe. Next you want to look for any retinal pathology, keep the gain at a level where the vitreous appears black. Now slowly fan back and forth looking for any irregularities along the retina. In our image you see a large retinal detachment floating in the vitreous and anchored at the optic nerve sheath. If you see a normal retina, slowly increase the gain while the patient moves their eye right to left. If you see any hyperechoic structures swirling within the vitreous this is likely a posterior vitreous detachment or vitreous hemorrhage. We will cover these in later posts. https://www.instagram.com/p/CJtPm8ZH8wu/
Clinical Case Cardiac transesophageal (TEE) Post ROSC RV dilation Right ventricular enlargement after ROSC, TEE views IVC clots, RV dilation EMS calls and they are in route performing CPR on an 85 year old female who was found down at home by family. The patient is intubated and in PEA arrest. CPR has been ongoing by EMS for 20 minutes and she has received multiple rounds of epi. On arrival to your ER while the LUCAS is being placed on the patient you are able to obtain the following images. What do you think about these images and what is your management plan moving forward? The charge nurse is checking the carotid and states that he does not feel a pulse. Are you going to resume compressions? -- Answer Below – Resume Compressions? – No Management plan – The ultrasound images show coordinated cardiac contraction. This indicates that return of spontaneous circulation (ROSC) was achieved at some point, likely recently given other findings. Even though the RN is saying he cannot feel a pulse the patient has what appears to be a perfusing rhythm on ultrasound. It can be exceedingly difficult to palpate a pulse in a high stress situation such as cardiac arrest especially if the patient is hypotensive or bradycardic. The next few steps in management should be evaluating perfusion status and stabilizing the patient. There is not a lot of evidence on how to tackle this exact situation but here is my approach: 1. Establish good peripheral access (large bore PIV, central line, or IO). 2. Empirically start Levo or Epi infusion while placing an arterial line (usually fem). 3. Begin evaluating for causes of arrest (send labs, perform RUSH exam, complete physical exam and any indicated imaging). Now there are a few other interesting findings in this ultrasound. You can see there is extensive turbulence and clot in both the arterial and venous systems, you can even see clot formed on the valve leaflets. This is likely from extended down time in the field. Some of you may have noticed that the RV size is abnormal and there is a D-sign present. While pulmonary embolism should be considered, especially in PEA arrest, these findings are common post ROSC. After about half an hour on an epi drip this patient’s RV size returned to normal and LV function improved. https://www.instagram.com/p/CJthgNTHe5_/
Clinical Case Lower extremity Musculoskeletal Pediatric hip effusion Normal hip effusion size, pediatrics joint effusion, hip effusion Pediatrics A worried mom brings her 5 year old son into your ED. She says that for about the past 3 days her son has been limping on his right side. She doesn’t remember him hurting the leg. He has no medical history. He refuses to weight bear on the right leg, but otherwise he is well appearing, playful and vital signs show a normal temperature. You perform a point of care ultrasound (POCUS) and see the above images. What is your diagnosis and how are you going to workup and manage this patient? -- Answer Below – Diagnosis: Hip joint effusion Management: In the ED evaluation of pediatric limp is common. When it comes to the hip there are multiple causes but one of the real can’t miss diagnoses in the ED is a septic joint. Now the child here appears well and is afebrile. However, kids are famous for looking fine even when they are sick. When it comes to pediatrics you should always have a high level of suspicion for badness even in well appearing kids. That being said, not every kid needs a major workup. A lot of times watchful waiting is the right answer in pediatrics, but here POCUS identified something that is concerning so we should go a step further. Our major differentials after seeing the POCUS should be Toxic synovitis (benign) vs septic arthritis. The next step should be to draw ESR and CBC. Using the Kocher criteria, (ESR > 40 or WBC >12K) we can further risk stratify our patient. This patient’s ESR and WBC both came back normal, Xray was normal as well. PO Motrin was given and the patient was running around the ED asking for snacks. A diagnosis of toxic synovitis was made and the patient was discharged with return precautions. https://www.instagram.com/p/CJwP_vkHe66/
Clinical Case Gallbladder Cholelithiasis with borderline common bile duct findings of cholecystitis, normal common bile duct measurements, gallstones, common bile duct (CBD) measurements You have a 42 year old female present with epigastric pain. You obtain the above point of care ultrasound (POCUS) images. Despite pain control the patient continues to be very uncomfortable and cannot tolerate PO. What is your diagnosis and what are your next steps in management? -- Answer Below – Diagnosis: Cholelithiasis with borderline common bile duct (CBD). Management: The ultrasound shows gallstones and no signs of cholecystitis. Remember gallstones and a dilated CBD are NOT signs of cholecystitis. Do you remember the 5 signs?.... 1. Thickened anterior GB wall >3mm 2. Pericholecystic fluid 3. Sonographic Murphy’s sign 4. Hydropic GB, which is a GB measuring >10cm long x 5cm wide 5. Non-mobile stone lodged in the GB neck So since this patient does not have cholecystitis your current working diagnosis is symptomatic cholelithiasis. Now the patient’s CBD measure 5.1mm, sources vary but for a good sensitivity, I use 4mm + 1mm for every decade of age >40 years old as normal. However, a CBD up to 7mm can be normal and CBD <10mm is normal after cholecystectomy. Given the patient’s intractable pain you should consider early choledocholithiasis as a possible diagnosis. This is a crucial diagnosis to consider because it can lead to ascending cholangitis, which is life threatening. If the patient’s lab work is normal she should be admitted for MRCP to confirm your suspicion. However, if the patient has abnormal LFTs, lipase or bilirubin, gastroenterology should be consulted for ERCP, which is both diagnostic and therapeutic. https://www.instagram.com/p/CJyeDu2nrq9/
Clinical Case Bowel Small bowel obstruction Findings of small bowel obstruction (SBO), plicae circulares A 63 year old female presents to your ED with 24 hours of intractable nausea and vomiting. She says over the past 4 hours she cannot drink a sip of water without immediately throwing up. She has a surgical history of appendectomy and no medical problems. You perform the above point of care ultrasound (POCUS). What is your diagnosis and management? -- Answer Below – Diagnosis: Small Bowel Obstruction (SBO) Management: Ultrasound is good at detecting SBO (sensitivity 97.7%, specificity 92.7%), much better than abdominal plain films (sens 75%, spec 66%). *US features below. Now in a patient with recurrent SBOs that have been managed non-operatively it is not unreasonable to stop at the ultrasound or plain film. However, we generally will get a CT scan. CT can identify the transition point, look for complete vs partial obstruction, identify less common etiologies (hernia or malignancy), and look for complications such as closed loop, ischemia, necrosis, or perforation. Beyond identifying the SBO management from an ED perspective is simple. Of course ABCs, focusing on replacing fluid losses and controlling vomiting; but if the CT does not show any complicating factors then the patient should have an NG tube placed after surgical consultation, made NPO, started on fluids, and admitted to sx service. There is some data to suggest that NG tubes are not useful in SBOs however everywhere I have worked sx wants them to be placed. I prefer that consultants like me… so I will generally ask if they want one and place it while the patient is still in the ED. The majority of SBOs will be managed non-operatively by our surgical colleagues. Now what do you look for on POCUS for SBO? First make sure you are looking at small bowel. While large bowel will contain haustra, the Jejunum has plicae circulares, which you can see in this clip, and ileum lacks haustra or plicae circulares. So once you see small bowel look for: 1. Dilated loops (>2.5cm in jejunum or >1.5 in ileum) 2. “to and fro” motion of the bowel contents 3. Lack of compressibility 4. Free fluid and bowel wall edema (>2mm thick) 5. Transition point Each element will increase your specificity https://www.instagram.com/p/CJ3_yxvnHFZ/
Clinical Case Cardiac Lung Congestive heart failure (CHF) Reduced ejection fraction, causes of CHF, CHF workup B-lines, reduced EF, pleural effusion A 58 year old female presents to your ED with worsening shortness of breath. She notes that her only medical history is diabetes and she has never had symptoms like this before. The Dr. caring for the patient asks you to evaluate for signs of pulmonary embolism. You obtain the above point of care ultrasound (POCUS) images. What is your diagnosis? -- Answer Below -- Diagnosis: Congestive heart failure (CHF) The cardiac images show a reduced ejection fraction with a normal appearing RV. The lung windows show bilateral B lines as well as pleural effusions. Together all of these findings suggest a diagnosis of CHF. While longstanding uncontrolled hypertension (HTN) is a common cause of CHF, in a patient that has no history of HTN other causes should be investigated. ED workup should include an EKG, looking closely for ischemic changes, as well as lab work include troponin and brain naturitic peptide. Patients with new onset CHF should be admitted for formal echo to evaluate for valvular pathology and possible cardiac catheterization. https://www.instagram.com/p/CJ7G9B5nd_W/
Clinical Case Lower extremity Musculoskeletal Patellar fracture Hematoma, fracture A 68 year old male presents to your ED after a fall. He has significant swelling of his right anterior knee and says that he is having trouble moving the leg. You perform the above point of care ultrasound (POCUS). What is your diagnosis and how are you going to manage the patient? -- Answer Below – Diagnosis: Patellar fracture with significant displacement. Management: Orthopedic surgery evaluation. Patellar fracture with >3mm of displacement are generally an indication for open reduction internal fixation (ORIF) and Ortho should be consulted from the ED. In the first few US images you can see that using the linear probe we were able to identify a large hematoma and a cortical irregularity within the patella on the right when compared to the left. There was difficulty identifying all relevant structures on the right using the linear probe so to get a wider view of the injury further images were obtained using the curvilinear probe. As you can see at this point, using the curvilinear probe, we were able to identify that the patella had fractured into two separate fragments. In the last clip you can see both the superior and inferior portions of the patella separated by a large hematoma overlying the distal femur. The last image is the lateral Xray of the knee. https://www.instagram.com/p/CJ9m0Tcnms2/
Clinical Case Gallbladder Renal Normal gallbladder with incidental hydronephrosis Look at the entire image, management of hydronephrosis hydronephrosis A 45 year old female presents with R upper quadrant and back pain. You perform the above point of care ultrasound (POCUS). What are your findings and how are you going to manage this patient? -- Answer Below – Findings: Normal gallbladder with significant right sided hydronephrosis. This ultrasound is a great example of why you need to pay attention to everything in your imaging window. You can see here that the operator was focused on imaging the gallbladder but incidentally picked up hydronephrosis of the right kidney. If you did notice the hydronephrosis then management could proceed as we have discussed previously with possible CT for stone vs discharge with close urology follow up. Now given the degree of hydronephrosis I would generally get a CT even if I were suspecting stone, however that is not always necessary. If you are not concerned about something other than a stone obstructing the ureter, and the patient has 2 kidneys, and the other one appears normal, it is generally safe to send them home even with significant hydronephrosis. Given all the other elements that we have discussed previously remain true such as urine without signs of infection and pain controlled in the ED. https://www.instagram.com/p/CKFFv9bhRlM/
Clinical Case Cardiac Bicuspid aortic valve Interesting case. This patient was being evaluated for shortness of breath. I was asked to do a bedside echo. I noticed that the aortic valve appeared calcified however upon closer inspection I suspected that the RCC and NCC of the aortic valve were fused. In the short axis view of the aortic valve it only appears as though the LCC is moving. I do not think this was what was causing the patients symptoms. I am interested if anyone has any thoughts? https://www.instagram.com/p/CKFH4YZBanc/
Clinical Case Cardiac Supraventricular tachycardia supraventricular tachycardia management pediatric heart, tachycardia, patent foramen ovale (PFO) Pediatrics A worried mother brings in her 2 week old son. She says that he has been acting off for the past few hours, has been extra fussy, and now he is not eating. The baby is hooked up to a monitor and the heart rate is noted to be 290. The remainder of the vitals are normal for his age. Exam shows strong peripheral pulses and no cyanosis. An EKG is obtained along with a bedside echo. What is your diagnosis and what are your next steps in management? -- Answer below – Diagnosis: Supraventricular tachycardia (SVT), infant rate usually >220 bmp Management: The first step in pediatric or adult cases that have the potential to go south is to make sure all necessary equipment is nearby. The baby should be hooked up to pads, have IV access established, and have airway equipment outside the room. (Vagal maneuvers can be attempted while accomplishing these steps). Given that this patient is hemodynamically stable and appears well perfused vagal maneuvers should be attempted first. Another major branch point is wide vs narrow QRS, our patient has a narrow QRS and the management of wide QRS tachycardia is different and not discussed here. If the patient is unstable then cardioversion should be performed immediately, the pediatric dose is 0.5 to 2 joules per Kg, synced for SVT. Vagal maneuvers in children include: diving reflex (Ice on face for 15-30 seconds), in older children you can ask them to Valsalva or blow into a syringe. If these maneuvers fail in a stable child the next step is adenosine. The initial dose of adenosine is 0.1 mg/kg IV (max 6mg), remember it is rapidly degraded and needs to be injected as close to central circulation as possible followed rapidly by a NS flush. Always remember to refer to your PALS algorithms for help in emergencies. Our child responded well to initial vagal maneuvers and quickly converted to a sinus rhythm. He was admitted for further evaluated by cardiology. As you can see from the echo the significant tachycardia is limiting functional cardiac output. This is one of the reasons that patients with SVT can be unstable. From the color flow images there also appears to be a PFO, which can be normal at this age. https://www.instagram.com/p/CKHLK11B6Lh/
Clinical Case Cardiac Normal diastolic function Diastolic assessment, pseudonormalization Diastology cheat sheet, tissue doppler, uptodate diastolic assessment A 64 year old female presents to your ED with 1 week of worsening shortness of breath. A chest xray is done and shows pulmonary edema vs atypical pneumonia. You consider CHF but note that her ejection fraction appears normal and wonder if she may have heart failure with preserved ejection fraction (HFpEF). Based on the above exam, what is your assessment of her diastolic function? -- Answer Below – The patient’s diastolic function appears normal. We have discussed the mitral valve inflow pattern in a previous post, but here again briefly. As the LV relaxes it creates a negative force that pulls blood from the atria into the ventricle. In the 2nd image, with the PW doppler over the mitral valve leaflets, this passive filling is the first wave you see and called the E wave. The next wave is the A wave and that corresponds to the atrial kick. Under normal circumstances the E>A wave. As the LV becomes stiff and fails to relax properly, the A>E, this is early diastolic failure or impaired relaxation. As diastolic failure progresses, the MV inflow patter becomes “pseudonormal.” The next (3rd) image above is a diastology cheat sheet from @gasmanjonny that helps explain this concept. As the LA pressure continues to increase due to longstanding diastolic dysfunction, once again the E>A wave due to high LA pressures causing greater early LV filling. So to differentiate between normal and “pseudonormal” you need tissue doppler (TD). TD is another mode on the ultrasound that specifically looks at the movement of tissues and can detect much lower velocities. The TD gate goes on the medial mitral valve annulus. Here in the 4th image you notice 2 negative deflections. These deflections once again represent LV relaxation and atrial kick and are called e’ and a’ respectively. As you can see our patient shows the normal pattern where e’>a’ and therefore has normal relaxation. This can be more complicated and I have include a flow sheet from UpToDate showing a more complete evaluation, but from an acute care perspective, if you are worried about diastolic failure, a normal E/A and e’/a’ pattern should reassure you. E/A = 1.61 E/e’ = 5.6 Septal e’ velocity = 13.88 https://www.instagram.com/p/CKNDcdTBhMF/
Clinical Case Gallbladder Acute Cholecystitis findings of cholecystitis, management of cholecystitis Thick gallbladder wall, stone in neck (SIN) sign, pericholecystic fluid A 24 year old male presents with acute onset RUQ pain that woke him from sleep. You arrive to your shift and he has been waiting for a RUQ ultrasound to be done for the past hour. He appears uncomfortable. You decide to do a point of care ultrasound (POCUS) to see if you can make a bedside diagnosis to help expedite his care. You obtain the above images, What is your diagnosis next step in management? -- Answer Below – Diagnosis: Acute Cholecystitis Management: After diagnosing acute cholecystitis you should reach out to your surgical colleges for assistance. While awaiting their consult, ED management should focus on symptomatic treatment and antibiotics. Even though acute cholecystitis is mostly thought to be an inflammatory process, secondary infection is possible and empiric antibiotics, while awaiting definitive surgical treatment, is still standard of care. Antibiotics should target gram negative rods as well as anaerobes and my normal regiment is Flagyl + Ceftriaxone (unless there are other complicating factors). Here in our patient, we see several signs of acute cholecystitis on ultrasound. Do you remember them all?..... https://www.instagram.com/p/CKSPpyPhhw6/
Clinical Case Lung Viral (COVID-19) Pneumonia Findings of viral pneumonia, COVID-19 findings Irregular pleural line, subpleural consolidations You are in the peak of COVID season. A 46 year old male walks in to your ED and he is hypoxic. You know that he is going to need admission. His general symptoms all align with COVID infection, but after the PCR comes back negative, you question if you should keep him on isolation precautions for admission. Unfortunately, the portable CXR machine is not working and radiology is not walking any COVID rule outs over for 2 views. You decide to perform a point of care ultrasound (POCUS) to increase your diagnostic certainty. Do you still think COVID is your most likely diagnosis? -- Answer Below – Most likely diagnosis: COVID The images above all paint a picture typical for an evolving viral pneumonia. You can see thickened and irregular pleural line at multiple points. Several fields show small sub-pleural consolidations (especially image 6). There are also large areas of poorly aerated lung with multiple B-lines along with scattered areas of normal aeration. The current ultrasound data regarding COVID suggests that posterior and inferior lung fields are involved in the majority of cases. Here we see that our patient has some B-lines even in the superior anterior fields, suggesting more severe disease and likely explaining his hypoxia. Notice also that there are no large pleural effusions. Pleural effusions are fairly uncommon in COVID so this would also support our diagnosis. Don’t forget that B-lines doesn’t always mean pulmonary edema. Multiple etiologies can produce B-lines. Therefore, it is always important to look for other subtle pleural findings that may suggest an alternative diagnosis and keep the overall clinical picture in mind. https://www.instagram.com/p/CKUX2Q7BXYc/
Specific Topic Aortic Arch Assessment Dissection Aortic arch evaluation, normal aortic arch sizes, dissection management Normal arch measurements, aortic arch view Dissection is almost always on the differential list when it comes to chest pain in the emergency room. Whether the medical student says, "does that mediastinum look wide?" Or the patient says, "the pain does go to my back a little." It seems like we can never escape the worry of thoracic aortic dissection. Even though this diagnosis is always on our mind, it is relatively rare. One paper from 2015 looked at 9.5 million ED visits and only found 782 dissections overall. (1) Out of those 9.5 million visits 763,000 (8%) were for atraumatic chest pain. So that means for every 980 atraumatic chest pains you see, one of them might have a dissection. Well a prospective trial from 2014 looked at bedside ultrasound in the diagnosis of dissection and found that dilation of the ascending aorta was present in 70% of dissection cases. (2) They used a cutoff of 4 cm on transthoracic echo at the aortic root to define dilation. As the aorta progresses it actually tapers down and the aortic arch averages about 2.2 to 3.6 cm. (3) Above you can see a parasternal long axis view showing a normal aortic root along with a graphic showing the average measurements along the path of the thoracic aorta. Another less common view you can get with POCUS to further evaluate the thoracic aorta is the suprasternal notch view. Most people I talk to say that they don’t use this view because it is too hard to get, the truth is, it’s really easy to get you just have to position the patient correctly. What does that mean....well the angle that we need the probe beam to be facing is very superficial in the chest, so the optimal patient positioning requires them to have their head out of the way. The easiest way to accomplish this is have the patient lay flat with a pillow underneath their upper back and their head against the bed. If you have an older patient who cannot tolerate extending their neck this much you can try having them extend as much as possible and side bend or turn to one side. https://www.instagram.com/p/CJRV54cHZlD/
Specific Topic Cardiac transesophageal (TEE) Pulmonary embolism TEE in the ED RV inflow outflow Transesophageal echocardiography (TEE) uses a specialized probe that is placed into the patients esophagus to get better images of the heart. In our department we only use TEE on intubated patients however TEE can be done as an outpatient with moderate sedation not requiring intubation. From an emergency perspective, TEE can be useful in identifying acute causes of cardiac arrest. Above the first image is a TEE view called RV inflow/outflow. As you can see this view would be invaluable in assessing for pulmonary embolism (PE) as the cause of cardiac arrest. You can see the right sided circulation all the way from RA to the pulmonary artery all in one clean view. During our resuscitation we went one step further to investigate PE as a potential cause of arrest and directly visualized the entire main pulmonary artery (MPA). Using standard transthoracic echo complete evaluation of the MPA is impossible. Here you can see we were able to trace the MPA all the way to the bifurcation into the right pulmonary artery. As TEE training becomes more common in emergency medicine it will undoubtably become a key tool in all forms of resuscitation. https://www.instagram.com/p/CKCAHU2nyKC/
Clinical Case Abdominal Aorta Aortic aneurysm Normal abdominal aorta Flank pain, anatomy of the abdominal aorta Celiac truck, superior mesenteric artery You are a 4th year medical student on your ED clerkship. You pick up a 72 year old male who is presenting for right flank pain. He denies any history of kidney stones and says that he has no prior history of pain like this. He denies any associated symptoms and his vital signs are normal. He does note that he has a history of smoking. Being an astute 4th year, you present the case to your attending and mention one life threatening diagnosis that you are worried about. What diagnosis is this and how could you use point of care ultrasound (POCUS) to help streamline your workup? Above are several images, can you identify the anatomy in each? What are the normal measurements? -- Answer Below -- Life threatening diagnosis: Abdominal Aortic Aneurysm (AAA) -- For a 5 minute video on how to perform a AAA scan check out A little cold gel on youtube-- The normal abdominal aorta measures less than 2 cm. 2-3 cm is considered ectatic and >3 cm is considered aneurysmal. Additionally any Aorta that fails to taper as it moves caudally is also considered abnormal. The first 3 clips show the proximal, mid, and distal measurement points for the aorta. The final clip is a longitudinal view of the entire aorta. The proximal aorta is identified in the first still image. This is classically called the seagull sign. To the left of number 4 is the celiac trunk coming off of the circular aorta to the right of number 1. The celiac trunk has 3 vessels that come off of it: the common hepatic artery (number 2), the splenic artery (number 3) and finally the left gastric, which normally cannot be seen. Above number 5 you can see the inferior vena cava (IVC). The mid aorta is identified in the second still image. The main way to identify the mid aorta is to visualize the superior mesenteric artery (SMA) (number 2) floating right above the aorta (number 4). The other relevant vessels in the area are the IVC (number 5) and you can see a small vessel that comes off the IVC running between the SMA and aorta, this is the left renal vein. You can also see the splenic vein (number 3) as well as the pancreas (number 1). https://www.instagram.com/p/CJXaWItHGIU/
Yes Clinical Case Lower extremity Vascular (DVT) Normal lower extremity venous scan Management of potential DVT lower extremity venous anatomy, ACEP DVT management It is 3 AM in a rural ED and a patient arrives and saying that she was sent in by her PCP because she is having unilateral leg swelling and pain. You don’t have any vascular ultrasound services at your hospital so you decided to perform a bedside deep vein thrombosis (DVT) scan. What are the necessary components for a bedside DVT scan in the ED and how do you manage the patient after the scan is done? Is the above scan negative or positive for DVT? -- Answer Below – The diagnosis of DVT on point of care ultrasound (POCUS) is made by identifying a non-compressible vein. The veins of interest for an ED 2 point DVT scan are: 1. The common femoral vein near the saphenofemoral junction 2. The popliteal vein near it’s trifurcation. The above scan is normal and does not show a DVT. The first image shows the relevant venous anatomy of the leg. --For a 5 minute video on how to perform ED DVT scans check out A little cold gel on youtube— As you can see in the first clip, the femoral-saphenous junction is identified on the right side of the screen. There is complete compression of the vein here so this is negative for DVT at this location. The second clip shows the 2nd compression point near the popliteal trifurcation where the popliteal vein divides into 2/3 vessels. These 3 vessels are the anterior and posterior tibial and the peroneal, however the peroneal generally arises from the posterior tibial, hence the 2/3. Now the key here is how to manage the patient with a normal DVT scan as seen above. The main issue is that unlike a vascular lab, in 2 point DVT scans we do not examen the small veins below the knee for DVTs. This more thorough DVT exam is generally done by a vascular tech since it is time consuming and generally lower yield. While below the knee DVTs are less likely to lead to life threatening pulmonary emboli, they are generally treated the same as above the knee DVTs. So since our patient may still have a below the knee DVT that was not seen, we should send a D-dimer to decide how to proceed. The last image above shows the American College of Emergency Physician’s (ACEP) clinical algorithm for DVTs. As you can see in completely low risk situations a negative D-dimer alone is sufficient to rule out DVT, however our patient has swelling and pain would not be considered low risk so a POCUS DVT scan should be performed. Now, as we can see, her scan was negative so next we need a D-dimer. If the D-dimer is positive her risk of below the knee DVT is high enough that a repeat ultrasound needs to be done within a week to look for propagation of a clot. Generally I send a D-dimer right at the patients arrival and once I am done with my POCUS exam the results are back and I can decide how to proceed. https://www.instagram.com/p/CJXetD8HVrH/
Procedure Internal jugular central line Transvenous pacer Central line insertion, transvenous pacing TVP in RV, wire in IJ Procedures Ultrasound guidance has dramatically increased the safety profile of many procedures over the years. Perhaps nothing is more well known when it comes to ultrasound procedures than central line (CL) placement. The first image is a temporary transvenous pacer kit, which is being placed in these clips. In image two you can see a CL guide wire running into the internal jugular (IJ) in cross section. The feared complication of IJ CL placement is a carotid artery injury. Now in image 3 you see a nice clean shot of the IJ in long axis with the guidewire traveling down at an appropriate angle. This is the MOST important step before dilating the IJ. A wire placed in the carotid is not ideal, but not particularly dangerous if recognized. Once you dilate the carotid artery you have created a big problem. ALWAYS confirm your guide wire is venous before dilating. In the final image you see a substernal view of the heart and a small metallic object in the RV. This is the final resting place of the temporary pacing wire. https://www.instagram.com/p/CJyo4koHA1k/
Procedure Nerve block Ulnar nerve block Ulnar nerve and adjacent anatomy Procedures A 64 year old man presents do your ED. He sustained several lacerations to his hand during a woodworking accident. He has a large laceration on the posterior portion of the 3rd digit, several palmar lacerations, and a 2cm laceration along the lateral side of his 5th digit. You want to provide adequate analgesia without subjecting the patient to multiple needle sticks along the margins of each laceration. What other options do you have? -- Answer Below – Answer: Ultrasound guided regional anesthesia Patients who present with multiple or complex lacerations that will require extended repairs are great candidates for ultrasound guided regional anesthesia. In the above patient ulnar, median and radial nerve blocks were each performed at the level of the forearm. The first ultrasound clip shows localization of the ulnar nerve just lateral to the ulnar artery. I added a still image to point out some anatomy. After the nerve is identified, in the second clip you can see a needle approaching the nerve through the flexor carpi ulnaris and hydro-dissecting the adjacent tissue. In the last clip you can see that anesthetic is being injected adjacent to the nerve and displacing it medially, this is an indication of optimal anesthetic placement. The patient required no local anesthesia for the duration of all laceration repairs. https://www.instagram.com/p/CJ1NddenP7h/
Statistics Likelihood ratios how to use a likelihood ratio, Fagan nomogram Likelihood ratios (LR) can be confusion. In our previous post we talked about some ultrasound signs for PE and their associated LRs. Now to use a LR you need to understand something called the Fagan nomogram. On the left is pre-test probability the middle are LRs and on the right post test probability. You draw a line from your pre-test probability through a given likelihood ratio and that will give you your post test probability. Generally a good + LR is 10 or greater and a good – LR is 0.1 or less. As you can see if you have a very low or high pre-test probability even a test with a great likelihood ratio won’t really change your post test probability. This is the reason we use decision rules like Well’s score and PERC to assess pre-test probability before ordering something like a D-dimer or CT angiogram. I would recommend checking out thennt.com and looking at their LR section. You would be surprised at just how useless many of the tests we order every day really are at changing our post test probability. https://www.instagram.com/p/CJrLLO7HfX6/
Meme Dogpack Meme https://www.instagram.com/p/CJwGXQGHBls/
Meme Did you find the baby? https://www.instagram.com/p/CJzEfgMHVxQ/
Meme A little cold gel https://www.instagram.com/p/CKB6hLHHYC-/
Meme Ultrasound IV https://www.instagram.com/p/CKMT6xFBs8o/
Clinical Case Cardiac Pulmonary embolism findings in acute pulmonary embolism McConnell's sign, dilated right ventricle You are working a busy night shift and a 46 year old female is brought back for shortness of breath (SOB). Vitals show a heart rate of 115, BP 119/70, pulse ox 96% on room air. She denies any medical history and she says that for the past 4 hours she has felt like her heart is racing. She says that she decided to come to the ED because it has gotten so bad that she cannot even walk across the room without feeling extremely short of breath. The patient is worried that she may have COVID because 2 of her friends were recently diagnosed with it. You perform a point of care ultrasound (POCUS). What is your top differential and what is your next step? -- Answer Below – Top Differential: Pulmonary embolism (PE) The above echo shows a significantly enlarged RV and septal flattening on the parasternal short axis view (D-sign). Although the apical views are somewhat limited, there does also appear to be a McConnell sign. Notice how the RV free wall appears to be severely hypokinetic while the apex is relatively mobile, this is McConnell’s sign. There are several proposed mechanisms for why this happens. The explanation that seems the most logical to me is that, overall the RV has acutely failed secondary to severe, acute, pressure overload, however the RV apex is being pulled in by the still functioning LV. In a healthy young person McConnell’s sign is very specific for PE. However, as you can imagine this can also occur in other causes of RV failure where LV function is preserved such as RCA infarct or longstanding pulmonary artery hypertension. While the overall data on McConnell’s sign regarding sensitivity and specificity is mixed, it is more commonly associated with massive PEs. Given these findings there was concern for massive PE causing acute RV strain. IV access was established, and the patient was taken directly for a CTA and a diagnosis of massive PE was confirmed. Thankfully the patient was at a large tertiary care center, PE response team was activated and the decision was made to take the patient for emergency mechanical thrombectomy. https://www.instagram.com/p/CKXSMcxB98U/
Meme Finally a touch screen for adults https://www.instagram.com/p/CKZBzWHBmFM/
Clinical Case Ocular Retinal detachment Macula on vs off management A 62 year old man presents to the ER complaining of flashing lights. He denies any history of vision problems but wears glasses for distance. You perform a POCUS. What is your diagnosis and what is the next step in management? -- Answer Below – Diagnosis: Retinal tear with small retinal detachment Next steps: A small retinal detachment should be dealt with urgently because it is likely not affecting the macula (Mac On). You should call your ophthalmology colleagues to arrange for prompt care (usually within 24 hours). If the detachment continues to spread and involves the macula then central vision becomes affected and final visual prognosis is more guarded. In these images you notice a very small detachment of the retina localized to one area. In one of our previous posts there was an example of a much more extensive retinal detachment. You can see here how when it comes to ocular ultrasound, you need to look closely for subtle findings that could represent serious pathology. The first image is a fundoscopic photo courtesy of our friends in ophthalmology confirming a small tear with a detachment. https://www.instagram.com/p/CKZbbQuBShJ/
Meme Berny waiting on the AP4 https://www.instagram.com/p/CKZi3xPh01Z/
Clinical Case FAST Spleen Subcapsular splenic hematoma and cysts interesting splenic cysts This is not so much of a clinical vignette as an interesting case. A 76 year old male presented with left flank pain. He said that he had a history of a large splenic cyst and is scheduled for an elective splenectomy. Vitals were stable with the exception of sinus tachycardia. A POCUS was performed to evaluate the spleen and the images are shown above. What do you see in these images? -- Answer Below – The spleen appears very heterogenous with multiple small pockets of fluid and debris. There is free fluid accumulating at the caudal edge of the spleen as well. Abdominal MRI from several years earlier had shown a large (13.7 cm) benign appearing splenic cyst. CT imaging from this visit was read as a large subcapsular splenic hematoma with regional mass effect. The patient was taken to the OR for splenectomy. Operative note describes a very large spleen with multiple fluid filled cysts some with active hemorrhage. After successful splenectomy the patient recovered well and was discharged with a diagnosis of benign splenic cyst with associated rupture. https://www.instagram.com/p/CKednc_BONe/
Procedure Incision and drainage Soft tissue Abscess Evaluate with ultrasound before draining A 34 year old male presents to your ED complaining of a painful area on his right lateral thigh. You examine the area and notice that it feels warm and appears slightly erythematous. You are unable to tell if there is a subcutaneous fluid collection. You perform a POCUS to help you decide. What are you going to do after seeing the above images and how will you manage the patient? -- Answer Below – Next Step: Incision and drainage (I&D) The above images show a small subcutaneous fluid collection. While there are multiple differentials for this, in the clinical scenario of a warm, erythematous area an abscess is the most likely diagnosis. I always recommended putting color doppler on the collection if there is any question. Additionally, light compression can elucidate “pussastalsis,” swirling of the liquid within the abscess, and can help differentiate it from a heterogeneous soft tissue mass. To perform an I&D first clean off the overlying skin. Next anesthetize the area. Needle aspiration does not work as well as I&D so make an adequate length incision to achieve this benefit. Poke down through the skin into the abscess cavity, now extend your incision, the length of incision should be about 2/3 the size of the abscess. Now manually express any drainage then use a curved hemostat to gently break up any remaining loculations. Both irrigation and packing are somewhat controversial. Personally, I do not irrigate and only pack very deep or large abscess. Routine antibiotics are also controversial. If the patient is febrile, immunocompromised, has significant surrounding cellulitis, or a very large abscess then antibiotics should be given. Beyond this I usually do not prescribe them. Two large NEJM articles from 2016 and 2017 showed less treatment failure in the antibiotic group (NNT ~14) vs I&D alone, however no change in morbidity in either group was seen. There was increased risk of diarrhea in the antibiotic group and as always increased risk of emerging drug resistance pathogens. Therefore, I see minimal benefit and some risks. I think this is an area where we could have a positive impact on antibiotic stewardship as ER docs. https://www.instagram.com/p/CKhdA2ih1Dy/
Clinical Case Obstetrics (OB) Nausea and vomiting Management of nausea and vomiting in pregnancy fetal heart rate using M-mode A 28 year old G2P1 female presents to your ED 16 weeks pregnant with complaints of nausea and vomiting (N/V). She denies any vaginal bleeding or lower abdominal pain. You obtain the above POCUS to evaluate the fetal HR. What is your subsequent management of this patient? -- Answer Below – Ultrasound: The images here show a normal appearing IUP with a FHR of 147. You may notice that M-mode is being used to evaluate the FHR. PW doppler has to potential to increase tissue temperature at the location of the PW gate, while M-mode does not carry this risk. Therefore, in the ED we tend to err on the side of caution and use M-mode to evaluate FHR. While many ultrasonographers will use PW doppler, it is not recommended for extended periods of time. As far as I know the risk of PW doppler is purely theoretical, however I still utilize M-mode for FHR measurements. Management: The treatment of nausea and vomiting in pregnancy has become somewhat more controversial in recent years. Zofran (ondansetron) was common for the treating N/V in pregnancy. However, in the last few years some literature suggested that Zofran may be associated with cardiovascular malformations and cleft palates. Subsequent large (1.8M pregnancies) studies have concluded that there is not an increased risk of congenital malformations. That all being said, since the concerns over Zofran started, Diclegis (Doxylamine + Pyridoxine) has become the standard 1st line treatment. Most algorithms suggest that if Diclegis fails then the patient should progress to dopamine antagonist (Reglan) and then if those fail a serotonin antagonist (Zofran). I usually find that by the time patients come to the ED they have already tried several types of medications so I end up discussing Zofran with them. I have also found that Diclegis is not covered by most patient’s insurance. If you can control the patient’s N/V in the ED, make sure that they can tolerate PO and that they have a plan for medications at home before discharge. If it is during daytime hours it is nice to speak with the patient’s OB to make sure you can both agree on a safe medication plan for the patient. https://www.instagram.com/p/CKmWKOJBf9e/
Clinical Case Renal Bladder Bladder mass indications for acute hemodialysis hydronephrosis, bladder tumor An 86 year old female presents with worsening right flank pain and abdominal swelling over the past 3 months. She states that she has been losing weight and doesn’t have much of an appetite anymore. She says that the amount she is urinating has decreased as well. What do you see in the above images and what should be your next step. What service needs to be consulted for this patient? -- Answer Below – Next step: CT abdomen and pelvis Consult: Patient will need admission with urology consult. The above images show moderate right sided hydronephrosis. The images of the bladder show a mass. The patient likely has a malignancy causing obstruction of the ureter on the right side leading to hydronephrosis. This is not a diagnosis you should make without a CT of the abdomen. This was done and confirmed that there was a tumor extending from the bladder compressing the right ureter. Urology needs to be involved in this case since the patient will likely need a nephrostomy tube or ureteral stent. The exact management is outside of the scope of emergency medicine, but in the ED you should focus on any acute issues related to renal failure. Generally if a patient has 2 functional kidneys and only one is obstructed, the body will compensate for awhile. While this patient did have an elevated Cr there were no acute indications for dialysis. Do you remember the indications for emergency dialysis? You can use the mnemonic AEIOU: A – Acidosis E – Electrolytes (Potassium usually) I – Intoxicants (Methanol, ASA, ect) O – Overload (fluid overload) U – Uremia symptoms (seizures, pericarditis, bleeding) https://www.instagram.com/p/CKpQZFkh6C3/
Basics Series Cardiac cardiac basics Basics Cardiac basics - cardiac is by far my favorite ultrasound to do. Even after you get good at it, finding great windows can be difficult. When starting out always remember that it is helpful to have the patient roll over onto their left side. This will generally make things a lot easier. Now the exact orientation of this view will change based on where you work. There are 2 key things to know. 1. Where is your indicator on the ultrasound screen. 2. Which direction does your probe indicator face. As long as you know these two things you can always orient yourself. In our department we keep the ultrasound indicator on screen left. There are 4 basic cardiac views that you should know. The first view is the parasternal long axis. We get this view with the indicator towards the patient’s right shoulder so it may look flipped for some of you. On screen left you see RV, LVOT, and LA all stacked on top of each other. In general these structures should be about 1:1:1, if you notice one is way bigger than then another, this should make you think something might be going on there. Next is the parasternal short axis view. Rotate the probe 90 degrees for this view and you will see LV and RV side by side. My favorite, but usually the hardest view to get is the apical 4 chamber. The indicator will be pointed towards the patients anterior chest/right side and the method I usually use to find an AP4 is to go just about 1 cm below the nipple in a male or just along the inferior fold of the left breast in a female and make small concentric circles until you see cardiac motion. Once you spot something, slow down and focus in on what you’re seeing. Here you can see RA/RV on screen left and LA/LV on screen right. Finally look for a subcostal view. To get this view place the probe just below the patients xiphoid process and shoot through the liver, pressing the probe tail flat against the patients abdomen, you will be able to see the heart. A key step here is holding the probe overhand so that you can actually press it flat against the patients abdomen. https://www.instagram.com/p/CKsDdfrBCQf/
Basics Series Lung Lung Basics Basics Lung Basics – Lung ultrasound is key in any acute care setting. I use the curvilinear probe for all my lung scanning. Start by placing the probe just below the patient’s clavicle on the right with the indicator facing towards the patient’s head. What you want to find is a location where you can see two ribs with the pleural line between and on either side. Swipe through the first few images to see what that looks like. Now after you have found your first location the next step is key….Stay still!! If you are sliding around you will either misinterpret or miss things. Keep your probe in one location and observe one or more respiratory cycles. First look closely at the pleural line. Does it look like ants marching or does it look static? If the line appears static this may represent a pneumothorax. You can switch to the linear probe to get a closer look and confirm whether or not there is pleural sliding. Next look in the “areas of interest” that I circled for key findings. Here is where you will find either A-lines or B-lines. Above I have examples of what each look like. A-lines represent normal lungs. B-lines represent some sort of pathology, most commonly pulmonary edema, however they can also be present in pneumonia, atelectasis and other pathologies. Keep in mind that you need at least 3 B-lines per field to call it pathologic. So now that you know what to look for, where do you look? I personally look at 3 zones per lung. First was anteriorly just below the clavicle. The next is lateral just below the arm in the anterior axillary line the last is the lung base. I do only 3 because of time constraints, keep in mind that you are essentially ignoring the posterior lung with this method. Many protocols describe 4+ zones per lung. At the lung base you are going to look for the same things you looked for before as well as pleural effusion. Slide the probe down the mid or posterior axillary line until you begin to see the diaphragm and/or abdominal organs. Above I have an image of both a normal lung base as well as one with a large effusion. Now repeat the whole thing on the other side! https://www.instagram.com/p/CKxgjMKhTd_/
Clinical Case Cardiac EKG ST elevation myocardial infarction (STEMI) STEMI management, difficult windows Cardiac ultrasound can be difficult! The above case is evidence of that. A 52 year old female arrives to your ED complaining of chest pain. Triage vital signs are normal and she is well appearing. You go to the bedside and attempt to obtain a cardiac POCUS while the EKG equipment is being brought over from another area. You are unable to get good views despite repositioning the patient several times. The EKG equipment arrives. What is your diagnosis and what are your next steps in management? -- Answer Below – Diagnosis: Inferior STEMI (STE in 2, 3, aVF with depression and TWI in aVL) Management: If you are at a PCI capable center then you should activate the Cath lab. If not, and transfer to a PCI capable center will take >90 minutes, then thrombolytics should be considered. The classic morphine, oxygen, nitro, aspirin (MONA) has mostly fallen out of favor. You should always give 325mg chewable aspirin as this is the only medication show to actually reduce mortality in MI. Otherwise, O2 is harmful if the patient is not hypoxic and nitro would be contraindicated in this case because of presumed pre-load dependance of the inferior wall MI. Now, morphine is associated with poor outcomes due to other treatment delays and likely not the morphine itself. However, there is a growing body of evidence that morphine is associated negative outcomes in STEMI. This case is just an example to remind you not to get discouraged if you have a hard time getting good views. Every patient is different and sometimes it is just really hard! The patient was found to have a 100% RCA occlusion. Post cath echo was also noted to be very difficult. Definity contrast (a version of contrast for ultrasound) was used to enhance the LV walls and they were still unable to comment on many aspects of the cardiac function. https://www.instagram.com/p/CKzhrDMhmpf/
Meme The guy she tells you not to worry about https://www.instagram.com/p/CK5aumSh3Un/
Clinical Case Bowel FAST Appendicitis Consider alternative diagnosis when the picture doesn't fit Pelvic free fluid A 56 year old male presents to the ED with worsening R sided lower abdominal pain all day. Pt says pain is sharp and has caused him to throw up several times. He notes that he saw a urologist recently and was told that he has multiple kidney stones on both sides. Denies fever or dysuria. You obtain the above POCUS images. What is your diagnosis and next step in management? --Answer Below— Diagnosis: Appendicitis Management: Surgical consult is the first step. If there is high clinical suspicion and the patient appears ill it may be appropriate to involve surgery prior to imaging. If there are features of complicated appendicitis (perforation, abscess, or phlegmon) on imaging, or if they are systemically ill, antibiotics should be started immediately (Zosyn). If there are no signs of complicated appendicitis, antibiotics initiation should be coordinated with the surgery team (generally Cefoxitin or Cefotetan). The reason you should wait and talk to surgery is if they are going to the OR the same day, the recommendation is to give the dose of prophylactic antibiotics within 60 minutes of first incision. Beyond antibiotics you should focus pain control and keep NPO for the OR. Remember, normal lab work DOES NOT rule out appendicitis. The patient went to the OR and diagnosis was confirmed surgically. My initial suspicion in this case was for kidney stones given the patients story so I began by scanning the kidneys and bladder. As you can see, in the bladder window there is free fluid. This tipped me off that my initial differential was likely incorrect. On the subsequent images, using the linear probe, you can see a blind ending pouch coming off of the cecum with some surrounding free fluid. Don’t get confused by the terminal ileum, which is in the same area and looks similar. You have to confirm that you are identifying a blind ending structure. What are the signs of appendicitis on ultrasound? Aperistaltic, non-compressible, dilated appendix (>6mm) Hyperechoic appendicolith with posterior shadowing Periappendiceal fluid collections Wall thickening (>3mm), mural hyperemia on color flow increases specificity https://www.instagram.com/p/CK-Ar6ZB-o-/
Basics Series Aorta Aorta basics Basics Aorta Basics – Scanning the Aorta can be difficult at first, but with a few key pointers you’ll be a pro! First use the curvilinear probe with the indicator facing the patient’s right side. Put the probe just below the xiphoid process. Identity the aorta by using the bright white curved line from the spine with the dark shadowing behind it. You should see the Aorta just anterior and towards the patient’s left. You will likely also see the IVC shaped like a teardrop to the anatomic right of the Aorta. There are three landmarks where you are going to measure the Aorta. The first is the celiac truck, next the superior mesenteric artery (SMA), last at the bifurcation into the common iliacs. The Celiac trunk comes directly off the aorta and usually branches into three vessels. On the ultrasound you will be able to see it branching left and right into the common hepatic artery and the splenic artery. This is referred to as the “seagull” sign. The other vessel that it gives rise to, the left gastric, is not normally visualized. Now slide down just a small amount and you will see the SMA. Knowing the structures that surround the SMA is important. You know you are the level of the SMA when you see the “clock mantel” sign, which is the SMA in cross section sitting just above the aorta. Notice here you have the left renal vein running below the SMA. This is where “nutcracker syndrome” takes place. If the SMA compresses the L renal vein at this position it can cause hematuria and flank pain. Above the SMA you see the splenic vein running into the venous confluence. This is important because this is all part of the portal system. The superior mesenteric, portal, and splenic vein all connect here. Overlying the splenic vein is the pancreas. Now keep following the aorta down until you see it split into the 2 common iliac arteries. Finally get a complete longitudinal view of the aorta. If any area measures > 3cm it is considered aneurysmal. https://www.instagram.com/p/CLDRSdVBd2V/
Clinical Case Gallbladder Cholecystitis with possible ascending cholangitis Dilated common bile duct You have a 61 year old male present with fever and epigastric pain. He has a history of recurrent pancreatitis and Etoh abuse. You obtain the above point of care ultrasound (POCUS) images. What is your diagnosis? -- Answer Below – Diagnosis: Cholecystitis with possible ascending cholangitis Management: The ultrasound shows several findings of cholecystitis. Do you remember the 5 signs?.... 1. Thickened anterior GB wall >3mm 2. Pericholecystic fluid 3. Sonographic Murphy’s sign 4. Hydropic GB, which is a GB measuring >10cm long x 5cm wide 5. Non-mobile stone lodged in the GB neck Notice the large "vessel" adjacent to the gallbladder. This was actually the CBD and measured out to 2cm, but was missed on the POCUS given how abnormally large it was. Also notice a thickened anterior GB wall, pericholecystic fluid and a hydropic gallbladder. At the time of scanning, no stones were identified, however there is gallbladder sludge present and a possible stone in the 2nd clip. Since acalculous cholecystitis is a rare diagnosis, especially in an otherwise healthy patient, a CT was ordered. The CT scan confirmed findings of acute cholecystitis however noted an associated pancreatic pseudocyst causing external CBD compression as well as associated pancreatitis. The radiologist suggested that GB findings represented pseudocystitis. Given the CBD obstruction with findings of cholecystitis and systemic symptoms such as fever, the diagnosis of ascending cholangitis must also be entertained. https://www.instagram.com/p/CLGRp3yBL1Q/
Clinical Case Bowel Small bowel obstruction (SBO) Findings of SBO Transition point A 34 year old F presents with worsening abdominal pain, nausea, and vomiting. She has a hx of hysterectomy 2/2 severe bleeding. She says that since this morning she has been unable to tolerate anything including water without vomiting. What is your diagnosis and next step in management? -- Answer Below – Diagnosis: Small Bowel Obstruction (SBO) Management: As we have discussed before, ultrasound is good at detecting SBO (sensitivity 97.7%, specificity 92.7%), much better than abdominal plain films (sens 75%, spec 66%). *Below we have ultrasound features of SBO listed. Usually we get a CT to help identify the transition point, look for complete vs partial obstruction, identify less common etiologies (hernia or malignancy), and look for complications such as closed loop, ischemia, necrosis, or perforation. In this case, the ultrasound actually caught the transition point! In these images you see decompressed ileum adjacent to a dialated segment with to and fro motion of the bowel contents. A CT confirmed severe ileitis causing stricture and SBO with transition point within the ileum. Beyond identifying the SBO management from an ED perspective is simple. Of course ABCs, focusing on replacing fluid losses and controlling vomiting; but if the CT does not show any complicating factors then the patient should have an NG tube placed after surgical consultation, made NPO, started on fluids, and admitted to sx service. Now what do you look for on POCUS for SBO? First make sure you are looking at small bowel. While large bowel will contain haustra, the Jejunum has plicae circulares, and ileum lacks haustra or plicae circulares, as seen here. So once you see small bowel look for: 1. Dilated loops (>2.5cm in jejunum or >1.5 in ileum) 2. “to and fro” motion of the bowel contents 3. Lack of compressibility 4. Free fluid and bowel wall edema (>2mm thick) 5. Transition point Each element will increase your specificity, with a transition point being the most specific. https://www.instagram.com/p/CLKjjp7hdkI/
Clinical Case Cardiac Transvenous pacer Symptomatic bradycardia Management of bradycardia TVP wire in RV An 84 year old female presents by EMS, they report that she was found down at home and family called 911. Based on the images above what do you think was going on, hint you have seen this before on here…. --Answer Below— Answer: Symptomatic bradycardia requiring transvenous pacing (TVP) The patient arrived with a HR in the 30s. She was placed on an epi drip with good response HR increase into the 60s. She gradually started to become less responsive to the epi drip. In the first few images you see the patient’s heart while she was first on the drip. In the Apical 4 you can see now the patient is significantly more bradycardic and you can see the tip of the pacer wire in the right atrium. The next 2 subxiphoid views show that the wire was successfully passed into the RV. Electrical and mechanical capture were achieved and the patient was taken to the cardiac intensive care unit for further care. https://www.instagram.com/p/CLNedTVBPCl/
Clinical Case Gallbladder Pancreatic tumor Extrahepatic CBD measurement Hydropic gallbladder, gallbladder sludge, CBD dilation A 67 year old female presents from her PCPs office with progressive painless jaundice for 2 weeks. You perform a POCUS upon her arrival. What do you notice in these images? -- Answer Below – The first few ultrasound clips show a gallbladder filled with sludge. At first I wasn’t even completely convinced that this actually WAS the gallbladder given how abnormal it appeared. However after identifying a few landmarks it seemed that this was the only possibility. Whenever you have trouble locating the GB on ultrasound there are a couple landmarks you can use to assist. You can visualize the right kidney in long axis and slowly slide your probe anterior along the inferior edge of the liver until the GB comes into view. The main lobar fissure (MLF) can be seen on POCUS as a bright band between the right and left hepatic lobes, it runs obliquely between the neck of the GB and the right portal vein. So if you can find the MLF you can track it back to the GB. Given how dilated the CBD appeared, a second image of the CBD was taken more medially. If you recall the “mid aorta” measurement location where you can see the splenic vein meeting up with the venous confluence, the other 2 vessels that connect here are the portal vein and the superior mesenteric vein. If you begin to trace the portal vein from here back to the liver, you will see the CBD exiting the liver just anterior to the portal vein. This portion of the CBD was also very dilated. A CT scan confirmed significant gallbladder enlargement, measuring 12cm and significant CBD dilation measuring up to 1.2 cm. There was no cause identified and patient is awaiting MRCP scan. https://www.instagram.com/p/CLXrel3hrWM/
Clinical Case Lung Pneumothorax Linear vs curvilinear probe, lung sliding Lung point A 47 year old female presents with worsening left sided pain and shortness of breath. Today she went for a left sided lung biopsy and has been having worsening pain ever since. You perform a POCUS exam. What is your diagnosis and next step in management? -- Answer Below— Diagnosis: Pneumothorax In these ultrasound images you can see why using a linear probe can be extremely helpful when looking for lung sliding. The first image with the curvilinear probe shows the pleural line but is difficult to see if there is lung sliding. Contrast that with the linear images where lung sliding is much easier to identify and you can clearly see it is absent on the left lung. In the last image you can see that there is a lung point, which is where you see lung sliding next to no lung sliding, this identifies edge of the pneumothorax. Management: Pneumothorax come in 2 major forms. Traumatic or spontaneous. Spontaneous pneumothorax classically occurs in tall skinny males, since Marfan’s is a risk factor. Other risk factors include Ehlers-Danlos, alpha-1-antitrypsin deficiency, smoking, as well as underlying lung disease with bullae/blebs. A traumatic pneumothorax occurs either when: 1. There is a wound that communicates with the outside world and the pleural space, or 2. Internal injuries such as rib fractures cause damage to the lung that allows air to escape into the pleural space. In this case, injury to the lung from the biopsy is likely allowing air to enter the pleural space and accumulate leading to a pneumothorax. Management of a pneumothorax depends on size and patient’s stability. Size is defined using measurements on the CXR. All patients should be immediately placed on supplemental O2 with a NRB as this increases the rate of pneumothorax reabsorption. If the patient is unstable then immediate needle thoracostomy followed by tube thoracostomy is indicated. Otherwise, the decision to place a chest tube or aspirate the pneumothorax will depend on the above factors and varies somewhat between institutions. https://www.instagram.com/p/CLXsU0yBUpE/
Meme Resident not taking clips https://www.instagram.com/p/CLXulnDhb9p/
Clinical Case Aorta Abdominal aortic aneurysm FAST can be inaccurate in AAA A 56 year old female presents with severe right flank pain onset several hours ago. She has a known history of abdominal aortic aneurysm up to 4.5 cm. She denies LOC and vital signs are stable. You obtain the above POCUS, what do you see? -- Answer Below – These images show a 4.2 cm AAA with minimal flow. In the power doppler images you can see that there is a small channel of flow below what appears to be a large saccular aneurysm. The patient underwent CTA that showed a stable AAA with a small channel of flow within it. The patient’s pain was found to be secondary to several renal infarcts that had developed as a complication of her significant vasculopathy. She noted that she had history of several peripheral arterial occlusions requiring bypass. Remember that many times diagnosing a ruptured AAA will be difficult with ultrasound. The Aorta is a retroperitoneal structure and many times a rupture will not result in a positive FAST. If you have a hemodynamically unstable patient and you identify a large AAA, this should be presumed to be the cause regardless of the results of your FAST scan. Remember the measurements of AAA? 1-2 cm = Normal 2-3 cm = Ectatic 3+ cm = Aneurysmal https://www.instagram.com/p/CLcF18FBv7R/
Clinical Case Cardiac transesophageal (TEE) Cardiac arrest An 86 Y M presents with ongoing compressions. EMS reports that family states pt had recent cardiac surgery and suddenly became unresponsive. TTE views are difficult with ongoing compressions and given recent sx, the decision is made to insert a TEE probe for further evaluation. Do you see any obviously reversible causes in this series of images? -- Answer Below – The first image shows the aortic valve in long axis with ongoing compressions. The patient’s anatomy was atypical making the classic TEE views difficult. Typically during CPR you want to identify the mid-esophageal long axis view where you can see LV and LVOT, here we only see the LVOT but it appears that compressions are shifted over the LVOT rather than the LV. Additionally, you can see the aortic valves opening independently from compressions, suggesting ROSC. However, compressions were paused and as you can see from image 2, there is agonal LV activity with significant wall motion abnormalities, decision was made to change position resume compressions and start patient on an epi gtt. Ongoing initial views on the TEE did not identify any obvious reversible cause such as tamponade or clot in transit. After further chart review it was noted that the surgery that the pt had recently undergone was an aortic root and aortic valve replacement. Once ROSC was again achieved you can see in images 3 and 4 that the surgically placed aortic valve appears to be in place and the aortic root also appears intact. Image 5 shows a severely dilated RA and a tricuspid valve with debris, as well as signs of stasis represented by the turbulent flow within both chambers. This is not atypical for post ROSC situations and should not alone be a trigger for pushing TPA. Overall there was no clearly identified cause of cardiac arrest on this TEE evaluation and patient was admitted to the ICU for further management. https://www.instagram.com/p/CLh1gERBHEh/
Clinical Case Lung Acute Bronchitis Antibiotic stewardship A-lines A 35 year old male presents to your ED with cough x 3 weeks. His doctor gave him a Z-pack at onset of the cough, but that didn’t help. Last week he was given steroids and Augmentin. He denies past medical hx. He has never smoked, never had fevers or shortness of breath. Vitals: HR 74, RR 16, BP 110/80, SpO2 is 98% on room air. Lung sounds are normal. You obtain the above POCUS images and CXR. What is your management and how are you going to counsel this patient. -- Answer Below – The ultrasound images show normal lungs with A-lines and no effusions, the CXR also appears normal. With normal vitals and physical exam as well no history elements to suggest pneumonia, a diagnosis of bronchitis should be made. Prior to the COVID-19 pandemic routine testing for acute bronchitis was not necessary, unless you suspected pneumonia. Abnormal vitals, abnormal lung sounds, or mental status changes would all be good reasons to suspect pneumonia. Cough in acute bronchitis is usually secondary to viral infection. The cough can easily last up to 3 weeks and productive cough or sputum purulence is due to sloughed inflammatory airway cells and does not indicate a bacterial etiology. Since acute bronchitis is caused by viruses, antibiotics won’t help. Oral corticosteroids have not been shown to be more effective than placebo for reducing severity or duration of cough in RCTs. Differential for Cough >3 weeks should include pertussis, especially if there is known positive contacts or coughing paroxysms with prominent posttussive emesis. However, none of that is present in this case. Unfortunately, patient expectations make many of us feel pressured to prescribe ineffective and unnecessary medications. A big part of our job as ED providers is patient education. Patient’s, especially ones who have been given antibiotics are worried that these medications are not working. It is important to always take your time and explain your thought process to your patients. I have found that many times just a five minute conversation and answering a few questions is enough to avoid unnecessary prescriptions. https://www.instagram.com/p/CLmu491hoRO/
Clinical Case Cardiac Wall motion abnormalities and LV thrombus Evaluating wall motion abnormalities (WMA) LV thrombus, A 54 year old male presents to your ED complaining of chest pain. He says that he has a long history of heart problems but cannot remember them all. He says that recently he was told he needed to take a blood thinner but stopped because he ran out. An EKG shows changes that appear similar to a prior EKG from 1 year ago. You perform a POCUS, what do you see? -- Answer Below – The above echo shows a moderately reduced ejection fraction with significant regional wall motion abnormalities (RWMA). Most notably there is significant akinesis of the apex with an associated LV apical thrombus. The easiest way to look for RWMAs is to know the major regional segments on echo and then visually observe each segment for myocardial thickening and thinning. The entire myocardium should have a similar amount of thickening and thinning during contraction and relaxation. If you notice any segments where you do not see this, you should suspect a WMA in that region. Now for ED purposes, know that RWMAs do not always mean acute ischemia. In addition to acute ischemia, old infarcts, scar tissue, or cardiomyopathies can all cause RWMAs. That being said in a patient where you suspect acute myocardial infarction and you identify a RWMA, especially if this region corelates to new EKG findings, you should treat that patient as an occlusive MI until proven otherwise. In this case given our patient’s history of being told he needed to be on blood thinners, presumably for the LV thrombus, and with an unchanged EKG, these RWMAs are likely chronic. The patient was started on a heparin drip and transferred to the CCU for further care. https://www.instagram.com/p/CLpX0pzBSj4/
Specific Topic IVC Fluid assessment How to use IVC for volume status IVC is garbage, Sorry have to rant here for a minute. I absolutely hate the IVC for assessing volume status. In 2011 Dipti published a meta-analysis concluding, “moderate level of evidence suggests that the IVC diameter is consistently low in hypovolemic status when compared with euvolemic.” This paper really kicked off the IVC craze. Looking at the studies they analyzed…out of the 5, 4 looked at trauma patients. The gold standard in those papers was “clinical dx of shock.” The raw measurements were all over the board. “Control groups” mean values were: 29, 11.9, 20.7, 13.4, 10.7 mm. “Shock” means were: 14, 5.6, 15.5, 7.7, 6.9 mm. While all papers agreed that “shock” had smaller IVCs, comparing raw values between papers, “shock” IVCs in some papers were “normal” in others. In 3, 10-13 was normal while 2 found that 10-13 meant shock. What about extremes? Less than 15mm means that they need fluid and > 25 means they don’t. 4 papers had means less than 15 in the shock group. Interestingly the one paper with a “shock” measurement mean > 15 was the only non-trauma paper. So even extremely flat IVC might only be valid in trauma. Collapse? Papers on vented patients say IVC collapse > ~16% predicts volume responsiveness. So what about in spontaneously breathing patients? One small study found that IVC collapsibility >40% had a PPV of 72%. A meta-analysis by Long in 2017 evaluated 17 studies and concluded “respiratory variation in IVC has limited ability to predict fluid responsiveness, particularly in spontaneously ventilating patients.” Plus, no one can even agree on where or how to measure the IVC! ASE says 1-2 cm from RA, other groups say just below diaphragm, at/distal to hepatic veins, some long axis some short. In 2010, Wallace found that measurements above the hepatic confluence did not correlate with other spots among healthy volunteers. This created the idea that you need to go below the hepatic vein to be accurate. Well a, big issue here is, Wallace found for “healthy” volunteers, at the diaphragm collapsibility range was 4-36%, hepatic vein 9-51%, and renal vein 13-57%. Remember >40% “is” abnormal, so how many of these “healthy” volunteers needed a fluid bolus? https://www.instagram.com/p/CLr4Yd8Blpe/
Clinical Case Upper Extremity Olecranon bursitis Elbow anatomy Swollen olecranon bursa, soft tissue edema A 46 year old male presents with worsening posterior elbow pain. Over the past few days the area has become, painful, swollen, red, and warm. He denies significant pain when moving the elbow but says that any pressure is very painful. Denies fevers or injury. You perform a POCUS, what is your diagnosis and management? -- Answer Below – Diagnosis: Olecranon bursitis Management: This patient has several features concerning for septic bursitis. The external exam was consistent with olecranon bursitis and the ultrasound shows “cobblestoning,” a feature of cellulitis, and an enlarged olecranon bursa. Rapid swelling, erythema, and tenderness should raise your suspicion for septic bursitis. You should have a low threshold to aspirate superficial bursa since these have a higher incidence of sepsis. However, deep bursa, such as iliopsoas, typically are not infected without systemic signs of infection and should not be aspirated in the ED. For non-infected olecranon bursitis, the treatment should be conservative with NSAIDs and RICE (rest, ice, compression, elevation) as well as PCP follow up and aspiration is not indicated. Presence of erythema over an infected bursa is NOT a contraindication to aspiration and aspiration should be done prior to antibiotics. Fluid should be sent for gram stain, culture, WBC w/ diff, and crystals. Generally leukocyte count will be >2000 cells/microL indicates infection, gram stain is positive in about 50% of cases. Culture is the gold standard. If aspiration shows <2000 leukocytes and gram stain is negative but there is high clinical suspicion for septic bursitis, then empiric antibiotics for staph and strep should be started while cultures are pending. MRSA should be considered if pt has a hx of MRSA, fever, or a prosthetic joint. In our patient there was a high clinical suspicion for infection given the extent of cellulitis on ultrasound as well as erythema and pain on exam. Aspiration showed leukocytes <2000 and a negative gram stain. He was started empirically on Keflex and we arranged for close ortho follow up in 48 hours. https://www.instagram.com/p/CL4_7PGhrfN/
Yes Procedure Ultrasound guided IV Peripheral catheter Propper ultrasound IV technique Inserting an ultrasound guided IV is a key skill for any acute care physician. That being said, there is a steep learning curve and starting out can be very frustrating. The video on my youtube channel goes more in depth on this but here are a few key pointers. First mistake I frequently see is trying to use the probe indicator to line up your shot. Making a mark on the patient is much more practical since even if your probe moves as you are grabbing supplies, the mark you made will not. The way that I do this, as you can see in the first clip, is I use the non-needle side of the angiocath to depress the patient’s skin over the target vein. Once I can see on my ultrasound that I am directly over the vein, I draw a line back through the ultrasound gel to mark my insertion path. Now I just use the Pythagorean theorem and look at the depth on the ultrasound to the vessel and then go back that distance from the probe and insert at a 45 degree angle. This assures I will hit the vessel exactly where I wanted to, avoiding any issues with the vein diving too deep or moving medially or laterally. The next key step is following your needle tip all the way down. First the to maximize success here you need to make sure the angle of your ultrasound beam is perpendicular to the needle. I do this by utilizing a lot of gel, tilting the probe back to line up my angle and then sliding up the arm toward the needle. Once I can see needle I rock the probe back and forth to maximize how bright the needle is. When I am happy with the brightness I lock in that probe angle in my hand and now I will only be sliding up and down the arm and the angle will not change. As you can see from the infographic you must be sure that you see needle TIP and not just shaft. To do this you need to have that angle locked in, see needle, slide away from the insertion site and see it disappear. Repeat this process all the way down the to the vessel, you should never see the needle while you are advancing. Always see it disappear and then advance into your field of view. Finally walk the needle down the vessel. This maximizes the amount of catheter inside the vessel before you attempt to thread it. https://www.instagram.com/p/CL64O4wBTSb/
Basics Series Gallbladder Gallbladder Basics Basics Gallbladder basics - first things first the GB can be hard to image. Start looking for the GB with a subcostal sweep. Place the curvilinear probe below the xyphoid process with the indicator to the patient’s head and sweep along the costal margin. If you can visualize liver, or even better the GB during this sweep then you should be able to get the rest of your exam here. The second image in the series shows how, with the indicator towards the patients head (note red circle on ultrasound image), you see the GB just below the liver. Since the GB lies obliquely, you will likely need to rotate a little left or right to get a nice full shot. If you are doing your costal sweep and cannot even see liver, like the 4th image, 1st try having the patient take a deep breath and holding while you repeat the sweep. If you see GB during this maneuver focus in on that area and manipulate your probe so that the patient doesn’t have to hold their breath during your exam. 2nd have the patient roll over onto their left side if they are able. Sometimes several breath holding maneuvers taking clips when the GB comes into view may be necessary. If you see the patient’s liver in these views but can’t find the GB, here are two tricks to help locate it. 1st move laterally and find the hepato-renal interface. Keep the transducer sagittal and slowly slide medially along the edge of the liver (6th clip). Generally, this will bring you straight to the GB. If that doesn’t work, attempt to locate the main lobar fissure (MLF), the bright band adjacent to the portal vein, this band will usually run obliquely between the neck of the GB and the right portal vein and you can trace this back to the GB. So what if despite breath holding and rolling, you still cannot even see liver? Well then, you will need to look between the ribs. Use the X minus 7 approach. Place your probe 7cm lateral to the patients xyphoid process. You will be looking through ribs but can generally visualize the GB (Pic 8). Slowly rotate the probe to eliminate as much rib shadowing as possible. You can also try switching to the phased array probe, which has a smaller footprint, to minimize rib shadows (Pic 9). https://www.instagram.com/p/CL7cGzKB3JI/
Meme My PCP sent me to rule out DVT https://www.instagram.com/p/CL9uD5sBlPA/
Specific Topic Cardiac output Fluid assessment cardiac output, velocity time integral (VTI) Cardiac output equation, LVOT VTI measurement Part 1 of 2 Deciding what intervention is best for a hypotensive patient can be hard. In the ED we tend to give a lot of fluids before we consider pressors or inotropes. I think we can do better and I think ultrasound can help! When we ask the question, “Is my patient fluid responsive?” What we really want to know is, will their cardiac output (CO) increase after we give fluids. CO = Stroke Volume (SV) x Heart Rate. On ultrasound we can calculate SV by looking at the left ventricular outflow tract velocity time integral (LVOT VTI). Here, SV = LVOT VTI x LVOT cross sectional area. Since the area should be constant in any single patient, we can say SV = LVOT VTI. To measure the LVOT VTI you need to get an apical 5 chamber view. The “5th” chamber is the LVOT and getting this view is fairly easy. When you have an apical 4, just tilt your probe so that the US beam points more anteriorly in the patient’s chest, this will open up that “5th” chamber. Next you are going to place a pulsed wave doppler gate in the LVOT at the level of the aortic valve. This will give you a negative tracing that represents your VTI. The measurement package on your ultrasound should allow you to trace the doppler waveform and give you a read out of the VTI. When the HR is between 55-95 then a normal VTI is 18-22cm. So if the patient’s HR remains relatively constant during our assessments then we can say CO = VTI, remember if the HR has changed significantly you need to take this into account when evaluating the CO. The values you are looking for are increase >15% with a fluid bolus or >20% if you are talking about ionotropes for cardiogenic shock. A lack of response could mean you are wrong about the type of shock, insufficient treatment (ie more fluids or meds), or futility such as end stage heart failure. See the next post for my thoughts on fluids and sepsis. https://www.instagram.com/p/CL-W5O9hmqH/
Specific Topic Fluid and sepsis LVOT VTI Starling curve and VTI Part 2 of 2 - If you don’t know how to calculate the LVOT VTI see the previous post. First take a look at the Frank Starling curve. Here we see that with normal contractility, VTI <18 likely portrays a situation where a fluid bolus will be helpful. However, you can see that if cardiac function is depressed, VTI <18 may need ionotrops rather than a fluid bolus, this is one of the key reasons that serial measurements are important. The grey area also represents a time when re-assessment is necessary. If you have a VTI of 19 and after 1 L of fluid and the VTI is unchanged, then you should re-evaluate your treatment plan. Now the purple circle on the FS curve is how I currently think about pressors and fluids in septic shock. This is only somewhat literature based and has a lot of my opinion thrown in, so take it with a grain of salt. The latest surviving sepsis recommendations changed from pressors only after 30cc/kg of fluids to within the first hour to maintain MAP >65. This still doesn’t really tell us how to decide what might be best for our patient. There have been several studies recently showing that early pressors are likely beneficial in septic shock. Now patient’s who have a VTI >22 and are hypotensive are in high cardiac output shock (classic distributive shock). These are usually young relatively healthy patient’s whose hearts are able to significantly increase CO to maintain organ perfusion despite their profound vasoplegia. If we look at hemorrhagic shock, while CO will compensate and maintain BP during the initial stages, we would generally agree that once hypotension develops, we anticipate seeing a decreased CO. This situation perfectly describes why sepsis is different. In sepsis, unlike hemorrhagic shock, the answer is not that the patient is losing fluids, but instead that they are profoundly vasoplegic. While fluids may temporize the situation, hours to days later, third spacing and fluid overload will likely result in iatrogenic harm. https://www.instagram.com/p/CL-j_ARBkF5/
Basics Series Renal Renal Basics Basics Start with the curvilinear probe. The right kidney is slightly inferior and more anterior compared to the left and this will be important to remember when acquiring your images. Start on the right by placing the probe in the mid axillary line in the lowest intercostal space with the indicator towards the patient’s head. You generally will see the kidney right away, but if not try sliding more anterior or posterior until it comes into view. You should be looking at the liver just in front of the right kidney in a long axis. When imaging the kidney, you want to identify the renal pelvis and cortex mainly. Take a look at the 3rd image. Here you can see on the top ultrasound the bright white pelvis and hypoechoic pyramids. However, the bottom ultrasound you really only see cortex and pelvis, which is more typical. The main reason I bring this up is to discuss hydronephrosis, which is back up of fluid into the kidney. Mild is classified as enlargement of the calices with preservation of the papilla (the base of the pyramids). Moderate shows rounding of the calices and obliteration of the papillae, often described as “bear claw”. Severe is characterized by caliceal ballooning with cortical thinning. Now take a look again at prior clip labeled rt kidney. Here we see that the collecting system in the renal pelvis doesn’t appear dilated, however you can see renal pyramids! The persistence of hypoechoic renal pyramids can be normal, especially in younger adults, and does NOT represent hydro if the renal pelvis appears normal. Look at the hydronephrosis chart and these 2 images at the bottom. This was one patient that had moderate hydro on the left and no hydro on the right. Notice the left you can see dilation of the pelvis while on the right it appears normal. The left kidney is higher and more posterior, so you want to place your probe on the posterior axillary line and just above the most inferior rib space. With the probe indicator towards the patient’s head you will see spleen just in front of the kidney. Make sure you image both kidneys in short and long axis. Finally, the bladder. Aim inferior into the pelvis and image in 2 planes. https://www.instagram.com/p/CMDO6B9h-Q5/
Clinical Case Lower Extremity Musculoskeletal Achilles tendon rupture Management of achilles tendon rupture, ankle assessment ruptured achilles tendon A 35 year old male presents with sudden onset left ankle pain while working out. Pt states that he was jumping and felt like someone hit him in the back of the ankle with a weight. After the sensation he was not able to walk on the foot without severe pain. You perform a POCUS exam of the left and right posterior ankle. What is your diagnosis and management? -- Answer Below – Diagnosis: Achilles’ tendon rupture Management: Achilles’ tendon rupture is often misdiagnosed as an ankle sprain and POCUS can be a key component when evaluating these cases. Our patient had a positive Thompson’s test (squeezing the effected calf does NOT cause plantar flexion of the foot), which made the diagnosis more obvious. However, in cases of partial tears the Thompson test may be normal and therefore the injury could easily be missed. Once an achilles’ rupture is diagnosed, ED management focuses on splinting and arranging close outpatient follow up. The patient needs to be splinted in resting equinus position (foot plantar flexed ~20 degrees). This position places the achilles’ tendon in optimal healing position by reducing the distance from the rupture site. Additionally, the patient should be told to remain non-weightbearing until they follow up with orthopedics. The above ultrasound images show a fairly normal appearing achilles’ tendon in the first image near its calcaneal insertion on the left. There is a small amount of fluid below the tendon just above the bone. The second and third clips are more proximal and show disruption of the left achilles’ tendon approximately 5 cm proximal from its insertion at the calcaneus. The last image shows a comparison of the same region of achilles’ tendon on the left vs right. Xrays of the ankle showed did not show any boney injuries. The patient was splinted and we were able to arrange close outpatient follow up with orthopedics for further evaluation and treatment. https://www.instagram.com/p/CMGMwGgh8Y8/
Basics Series Obstetrics (OB) Gynecology (GYN) OB Basics Basics With the curvilinear probe, start by placing the transducer just above the patients pubic bone with the indicator pointed towards the right. Once you can see bladder you will notice either vaginal stripe or uterus just posterior to the bladder. Rotate the probe 90 degrees towards the patient’s head. Identify the bladder and then the uterus in long axis. Note the vaginal stripe just posterior to the bladder; track that up into the cervix and finally the endometrial stripe. The endometrium is where you should be looking for an intrauterine pregnancy (IUP). Make sure that you always have the Pouch of Douglas in view since this is the most dependent area in the female pelvis and a common location for free fluid to collect. The uterus can also be in different positions in relation to the bladder. Occasionally you will see the fundus in front of the bladder and sometimes the fundus is retroflexed away from the bladder. Let’s talk a little about pregnancy. There is no well-established cut off for what level of beta HCG should guarantee that you see something on ultrasound. On tests you might see 1500 for TVUS and 2000 for transabdominal. However, this is outdated and shouldn’t even be considered in the real world. More recent literature has found that no bHCG level reliably RULES OUT normal intrauterine pregnancy if one is not visualized on US. Additionally, many ectopic pregnancies have very low bHCGs, so as an ER doctor; the number shouldn’t matter to you. The first thing you will see in early pregnancy is a gestational sac. This cannot be called a definitive IUP. That is because a pseudo gestational sac can be formed in ectopic pregnancies. The first LIKELY sign of an IUP is the double desidual sign. However, most people will not definitively call an IUP based on this since it is somewhat subjective, therefore the first DEFINITIVE sign of an IUP is a yolk sac. The next step in progression is a fetal pole. Now early this is a small hyperechoic bundle of tissue adjacent to the yolk sac. As it progresses it becomes a fully formed fetus. The fetal heart beat usually becomes visible around 6 weeks and is at the stalk of the fetal pole adjacent to the YS. https://www.instagram.com/p/CMKsYZmBPra/
Echo Cases Cardiac Normal Echo Assessment for pulmonary embolism, TAPSE, LVOT VTI, LV to RV ratio Trace tricuspid regurgitation Echo Cases Here is a case to test your echo knowledge! You were asked to come and evaluate a 25 year old woman. The ED team is concerned that she is having postpartum cardiomyopathy due to shortness of breath and leg swelling. She has a HR of 78, BP is 147/70, saturation is 90% on room air. The lung ultrasound shows minimal bi-basilar B-lines and very small bilateral pleural effusions. You perform the above echo. Do you think this is postpartum cardiomyopathy? Is there anything else you are concerned about now? Try and look through all the images and see what you think. -- Answer Below – Overall, this patient’s echo appears very normal. The LV EF is >55% and she has a normal VTI, which remember from previous posts means her CO is normal as well. These 2 findings go against a diagnosis of postpartum cardiomyopathy. Given the very minimal lung findings and the patient’s degree of hypoxia we were concerned that PE may have been the actual diagnosis. On the parasternal short axis view the RV does appear to be slightly enlarged, however it appears normal on the parasternal long axis and the apical 4. TAPSE is normal, there is no McConnel’s sign, and only trace tricuspid regurgitation. These findings would all suggest that IF a PE is present it is not affecting the RV. After performing our evaluation, we discussed our findings with the team. We felt that the top 2 differentials should be PE or a primary lung pathology causing the hypoxia. The team elected to perform a CTA to evaluate further for PE. The scan was negative for PE and the patient was admitted with a presumed diagnosis of hypoxia 2/2 basilar atelectasis and is being further evaluated by the inpatient team. Previous posts covering these topics. VTI 3/3/21 TAPSE 1/5/21 https://www.instagram.com/p/CMMw1KHh5UE/
Basics Series Ocular Ocular basics Basics Ocular basics – Ultrasounding the eye is easy and incredibly helpful in the ED. Since most of us don’t perform dilated exams, ultrasound is our best bet at diagnosing retinal or posterior segment pathology. The only real contraindication to ocular ultrasound is a suspected ruptured globe. Otherwise I think POCUS should be a standard part of our ED eye exam for most complaints. I prefer the tegaderm method. This is where you place a small tegaderm over the patient’s eye prior to applying the ultrasound gel. Either way make sure you use a good amount of gel because you want to apply minimal pressure to the eye. Place the indicator to the patient’s right and slide the probe up or down until the eye appears in the maximum diameter. Here you should see the anterior chamber and the lens. You may need to fan the probe back and forth a little to get all the structures in view. Keep in mind that the posterior layer you can see here includes retina, sclera and choroid so pathology of any of those layers may be seen here. The main locations you are going to be looking for pathology are in the vitreous and along the retina. Debris within the vitreous can be difficult to see so you should have the patient move their eye back and forth as you slowly turn up the gain. This will accomplish 2 things. First, it is easier to see debris that is out of view, since it will swirl into view as the eye moves. Second, the eye is subject to a lot of artifact so anything you see in the vitreous you need to confirm that it moves with the eye to assure that it isn’t just artifact. The optic nerve can be seen posterior to the globe as a linear hypoechoic area. You can measure the optic nerve when evaluating for elevated intracranial pressure (ICP). This measurement should be taken 3mm behind the globe and ideally when you see the nerve sides parallel to one another, since it can be artificially increased otherwise. A measurement >5mm has a sensitivity of 90% and a specificity of 85% for detecting ICP >20 mmHg. https://www.instagram.com/p/CMTF6giBYjV/
Clinical Case Bowel Diverticulitis Findings of diverticulitis Sigmoid diverticula A 57 year old female presents with LLQ pain x 10 hours. She denies fever, nausea, vomiting. She says that she has had this pain before and was told it was diverticulitis. She has no other medical history. You perform a POCUS exam, what is your diagnosis and how are you going to manage this patient? -- Answer Below -- Diagnosis: Acute Sigmoid Diverticulitis Disposition: Uncomplicated diverticulitis should be managed outpatient with antibiotics and GI follow up. There is currently some debate about whether uncomplicated diverticulitis even needs antibiotics. In this ultrasound image you see the dark wall of the bowel and small outpouching representing a diverticula. Bowel lumen is usually air and fluid filled and can look like mixed dirty shadowing or just a strip of bright white. You can also see a segment of hyperechoic fat behind the bowel that is consistent with localized inflammation. The findings of diverticulitis on ultrasound are: -A hypoechoic peridiverticular inflammatory reaction -Mural and peridiverticular abscess formation with or without gas bubbles -Bowel wall thickening (segmental mural thickening greater than 4 mm) at the point of maximal tenderness -Presence of diverticula in the surrounding segments If you are at all concerned for complicated diverticulitis then the patient should get a CT scan since ultrasound isn’t great at ruling out complicated diverticulitis. But our patient has less than 24 hours of symptoms and know hx of diverticuli with isolated LLQ pain, if she is tolerating PO and has a reassuring abdominal exam I would suggest discharging home with antibiotic RX and strict return precautions for worsening symptoms or fever. https://www.instagram.com/p/CMXRYbbhHAa/
Clinical Case Soft tissue Lymph node Full evaluation before performing an incision and drainage To cut or not to cut, that is the question. Here is a case of a soft tissue swelling just below a patient’s left jaw. Now given the location, the treating physician was not considering cutting into this since they already believed that it was a swollen lymph node. However, there have been many a time when I have considered cutting into something, only to look with the ultrasound and find that the “abscess” in question was either a lymph node or something else I shouldn’t be slicing into. Now even if you look with an ultrasound, lymph nodes are one of the things you still might mistake for an abscess if you aren’t careful. The above images show a enlarged, reactive lymph node in the neck with the typical vascular pattern. Sometimes lymph nodes can appear more hypoechoic. If you don’t put color flow on them, you might be even more tempted to I&D. In general, if you are using ultrasound to help you decide if you should I&D a potential abscess, I have the following 3 recommendations: 1. Measure depth and size of the “collection” you are going to be cutting into, this way if you have already cut deep enough and aren’t getting any purulent drainage you can re-evaluate. 2. Image in 2 planes to confirm that it is blind ending. If it connects to a vessel or runs off deep, then it probably isn’t something you should be trying to drain at the bedside. 3. Put color flow on it. Color flow can help to show you that the structure you are looking at is not an abscess and instead is something with vascularity. A little more about lymph nodes. It is good to know some normal and abnormal features. Reactive lymph nodes can become exceptionally large and may even be larger than their malignant counterparts. Benign, non-reactive, lymph nodes can have sharp OR blurred borders. Presence of an echogenic hilum, as in this case, is typically a sign of benignity. Normally a lymph node will demonstrate hilar flow (as seen here), which is described as flow branching radially from the hilum originating in the cortical region. https://www.instagram.com/p/CMdWaFyhYLi/
Yes Clinical Case Testicular Testicular torsion Color flow in torsion, resistive index Torsed testicle with reactive hydrocele Pediatrics A 14 year old male presents with sudden onset left scrotal pain this morning that woke him from sleep. He denies ever having this pain before. He has associated nausea and vomiting and appears very uncomfortable. You perform a POCUS, what is your diagnosis and next step in management? -- Answer Below – Diagnosis: Testicular torsion Management: Emergent urology consult for operative detorsionIf urology is not immediately available manual detorsion can be attempted using the “open the book” technique. Note that about 1/3 of the time, the testicle is torsed the other direction. I think that testicular ultrasound is one of the modalities that people are a little more hesitant to learn. Depending on how you use it, testicular ultrasound can be very easy. In general the structures you are looking at are superficial, easy to find, and easy to image. I think the hesitancy comes in terms of being able to interpret the images. I will post a testicular basics at some point, but I think that this case demonstrates just how easy and useful POCUS can be in testicular emergencies, specifically torsion. As you can see here, even with color flow turned up significantly, there are no identifiable vessels in the left testicle. Compare that to the right where you clearly have flow. This is the most basic form of looking for torsion and I think this is how most ED physicians should be using POCUS, as a quick rule in. Now if it appears “normal” then the patient should undergo formal US, unless you are comfortable with a more advanced assessment. As in this case torsion was diagnosed simply by observing NO flow in the left testicle. Making this diagnosis within minutes of the patient’s arrival can be very beneficial. The quoted rates of salvage are <6 hours since onset ~ 100%, 6-12 hours 50% and 12-24 hours 20%. A more complete US would include ID arterial and venous waveforms. Other signs of torsion would be: the “whirlpool sign,” which refers to a lamellated mass with just cephalad to the testicle representing the coiled spermatic cord, as well as increase in size of the testicle, heterogenous texture, and a reactive hydrocele. In incomplete torsion, a resistive index >0.75. https://www.instagram.com/p/CMfjjg9Bd6X/
Clinical Case Ocular Valsalva retinopathy with possible choroidal detachment Posterior eye ultrasound can diagnosis more than just retinal problems A 54 year old female presents with complaints of vision changes in her right eye. She states that suddenly after sneezing this morning she sees “blood swirling in front of her vision.” She admits to mild pain in the eye. On exam 20/70 in the effected eye and 20/20 in the other. Bilateral IOPs are normal. You perform a POCUS to evaluate the posterior segment. What do you see and what is your diagnosis? -- Answer Below – Diagnosis: Valsalva retinopathy with possible choroidal detachment (Reasonable if you said retinal detachment (RD)) So, this ultrasound shows a bright white hyperechoic semi-mobile linear structure in the posterior segment. It appears that this anchors to the back of the eye and therefore on our initial evaluation ophthalmology was consulted with a concern for RD. The ophtho resident got some assistance from the retina fellow since the dilated exam was somewhat difficult. Their conclusion was that this likely represented valsalva retinopathy, but a choroidal detachment could not be ruled out. This is a diagnosis I have never even heard of, but apparently is somewhat common in the ophtho world. It is associated with a sudden rise in intraabdominal pressure against a closed glottis (hence the valsalva) and in this case likely caused by the repeated sneezing. It is rupturing of the superficial retinal vessels leading to pre-retinal hemorrhage below the internal limiting membrane. This makes sense for how it appears on our ultrasound! That all being said, this case is a great reminder that not all that appears like an RD is one! A posterior vitreous detachment (PVD) can also look like an RD and there are several features to help distinguish the two. When the eye is moving an RD will stay anchored and may “wave” around in the eye, while a PVD will usually swirl around freely if it is completely detached. Choroidal detachment is much less common than a PVD and looks almost identical to an RD. One feature that can distinguish the 2 is that an RD should terminate before the margin or the optic nerve sheath or just at its edge, while a choroidal detachment may anchor within the margins of the nerve sheath. https://www.instagram.com/p/CMk0--8hi7B/
Clinical Case Bowel Large bowel obstruction Transition point from the small to large bowel Amazing images caught by some of the residents. This patient is an elderly gentleman that came in for abdominal distention and pain. He had severe dementia and could not answer many questions. POCUS captured the above images. Can you tell what you are looking at here? What do you think is the diagnosis? -- Answer Below – Answer: Large Bowel obstruction The POCUS images show a distended segment of large bowel with mildly dilated adjacent ileum. In several of the images you can see the ileocecal valve intermittently allowing contents into the cecum. You can see swirling of the contents within the colon and minimal peristalsis which is typical of the large bowel. This is the first time I have ever seen something like this on ultrasound and would love any other thoughts or comments from experts on bowel ultrasound. An abdominal CT demonstrated a colonic obstruction with an abrupt transition in the distal transverse colon. Overall amazing case and kudos to the residents who captured these incredible images. https://www.instagram.com/p/CMpX24-hUVI/
Basics Series Knobology Knobology Basics Basics At first approaching an ultrasound machine can be intimidating. Most have keyboards with a TON of buttons. However, for most basic POCUS scans there are really only 6 buttons you need to press! Using the M9 keyboard as an example, first you should identify a button usually labeled patient or ID. Click this to input the patient’s MRN so that your images save correctly. Next click the probe or exam button. This will bring up a screen to select your probe and exam type (FAST, OB, ect). Choosing the correct exam optimizes certain settings and features for that exam. Additionally, the exam type will dictate the measurement packages available during your scan. After your probe has loaded in, the next button you should click is usually labeled B or 2D, the B stands for brightness mode. This is the standard 2D imaging. As you can see on our M9 the other buttons in that row represent different imaging modes. (C = color, PW = Pulsed wave, M = M mode). We will use the bladder window as example images. Now, grab the probe, put a little cold gel on it, and place it on the patient. First if your machine has a TGC (time gain compensation) reset this to default with all dials in the middle. The TGC increases gain at specific depths and should only be adjusted as needed. Once you have a view, identify any relevant structures in the near/far field and adjust your gain so that you can clearly make out the structures you are looking for. Ideally the gain should be adjusted so areas that are fluid filled show a nice, anechoic black. Other tissues should appear grey/white and you shouldn’t see any super bright white unless you are imaging bone. Next is the depth adjustment. Optimizing your dept actually maximizes your resolution because of how the ultrasound probe works so ideally at LEAST 2/3 of your screen should be useful to what you are trying to visualize. Using the bladder here as an example the far field should only really contain enough to verify there is no fluid in the rectovesical space. Finally, identify the save/still button and the clip/cine button. You will use these to save clips as you scan. https://www.instagram.com/p/CMuyckkBv8y/
Meme Sister Jean https://www.instagram.com/p/CMu4ZxMBq1e/
Pathology Series Cardiac Pericardial effusion Cardiac tamponade Pathology So you see a pericardial effusion on POCUS, the patient has a borderline blood pressure and is mildly tachycardic. Is this developing tamponade or something else? In this first post of the pathology series I will cover the ultrasound features of tamponade. There are 5 features of tamponade on POCUS 1. Pericardial Effusion 2. Systolic RA collapse (earliest) 3. Diastolic RV collapse (highly specific) 4. Plethoric IVC or R IJ @ 90 degrees 5. MV or TV “pulsus paradoxus” First you need to identify a pericardial effusion. This will appear as an anechoic or hypoechoic stripe between the pericardium and the heart seen in multiple views. A few things are frequently mistaken for a pericardial effusion that aren’t. On the parasternal long axis (PSLA) you may see a hypoechoic region in front of the heart, if this hypoechoic stripe isn’t seen behind the LV then it is likely a pericardial fat pad and NOT an effusion. Additionally, if the fluid is posterior to the descending aorta on this is a pleural effusion. Identifying systolic RA collapse can be difficult. The best way I have found is to get a good apical 4 and click the freeze button. Then you can scroll back and observe the walls of the RA when the TV leaflets are closed. To see RV systolic collapse I personally use a similar method, but many utilize M-mode here. If you can get either a PSLA or subx view that allows you to drop an M-mode cursor through the RV free wall and the MV at the same time you can see the RV free wall collapsing right after the MV opens. Most agree that IVC >2.1cm and <50% respiratory variation is fairly sensitive for tamponade (~95%), but has poor specificity (~40%). I also use IJ at 90 degrees if I am having trouble getting the IVC views. Finally “pulsus paradoxus.” Here what you are looking for is either MV or TV inflow variation with respiration. Get an AP4 and place a PW doppler gate at the opening of either valve and record several beats asking the patient to take a deep breath. Variation of the MV >25% or the TV >40% represents sonographic pulsus paradoxus. At the end of the day tamponade is a clinical diagnosis. Large effusion + hypotension and tachycardia = treat like tamponade. https://www.instagram.com/p/CMx13JjhUaa/
Yes Basics Series FAST FAST Basics Basics The focused assessment with sonography for trauma (FAST exam) is essential for every acute care provider. It can identify >500cc of hemoperitoneum in trauma. In a supine adult the most dependent portion of the abdomen is Morrison’s pouch in the RUQ. This is where you should start. Place the probe, indicator towards the patients head, just below the costal margin, midaxillary line. To call the RUQ negative you need to visualize the interface between the kidney and the liver and the most inferior edge of both. Carefully evaluate any anechoic stipes. To avoid false positives, observe for peristalsis, imaging in 2 planes, and applying color flow. Next, identify the spine and diaphragm. Normally, the spine will abruptly end at the diaphragm. US beams do not travel well through the aerated lung and therefore cannot visualize the continuation of the spine above the diaphragm. A pleural effusion (hemothorax in trauma) allows the US waves to travel through the liver, diaphragm, hemothorax, and then you will see the spine above the diaphragm. Next the LUQ. Remember “knuckles to the bed,” these windows are more posterior and superior so the probe should be placed on the POSTERIOR axillary line in the lowest intercostal space. Fluid collects above the spleen first so in addition to the same areas as the RUQ, the entire interface between the spleen and diaphragm needs to be evaluated. Next go just above the pubic bone with the indicator towards the patient’s right, aiming deep into the pelvis until you see bladder. Image in 2 planes. Since other pelvic organs in both males and females can appear hypoechoic, use the same tricks as above to avoid false positives. Edge artifact from the bladder might give you anechoic areas on the sides in a transverse view. If you rotate and do not see these in the sagittal plane then you know it was a false positive. Generally, if there is free fluid adjacent to the bladder you will see loops of bowel clearly rather than haze. Finally obtain the subxiphoid cardiac (not shown) looking for pericardial effusion (hemopericardium in trauma). If you are having trouble you can always look using another cardiac view such as parasternal or apical. https://www.instagram.com/p/CM0HvZphnL-/
Clinical Case Cardiac Gallbladder Sepsis of unknown source TTE is not good at picking up valvular vegetations A 60 year old male presented after a fall at home. He has a history of aortic valve replacement, HTN, and alcohol abuse. Vitals axillary temp 98, HR 109, BP 105/80, Sat 98% on RA. He complains of worsening fatigue and weakness, as well as chills over the past week. He had imaging ordered of his back and basic lab work. Labs pending pt was signed out to the oncoming shift. RN called team to the bedside because patient was acutely SOB with diaphoresis. Repeat vitals now oral temp 102, HR 140, BP 80/40, Sat 90% on RA, severe tachypnea. POCUS performed, images shown above. Patient was started on levophed and broad spectrum abx for presumed septic shock of unknown origin. POCUS shows reduced EF however with tachycardia difficult to quantify. Aortic root enlargement consistent with prior echos, difficult to visualize aortic valve but color flow shows no severe abnormalities. BL B-lines worse in the R apex, somewhat more consistent with PNA rather than edema, pt was started on BiPap for WOB with possible cardiogenic pulmonary edema. RUQ showed hydropic gallbladder with thickened wall, normal CBD, with no pericholecystic fluid and no stones. Full set of septic labs were sent off. Now there is concern that lungs, GB, or back pain could represent source, dedicated lumbar CT and CT chest and A/P ordered. Labs returning with WBC in the 20s with significantly elevated neutrophil:lymphocyte ratio. Lactate returns at 5. CT abdomen changed to CTA to look for ischemic component. Despite high dose levophed pts BP not improving much, vasopressin added, thiamine added, thought to be contributing to lactic production given EtOH abuse hx. Metabolic acidosis with a pH WNL with respiratory compensation. HR and BP now improving and repeat echo shows EF~ 50%. CT results show BL ground glass opacities in the lungs, multiple arterial thrombi including SMA and hepatic artery, hydropic gallbladder without other signs of cholecystitis, no changes in the lumbar spine. Patient stabilized and admitted to MICU with vascular surgery consult and started on heparin gtt. Inpatient TTE showed same as POCUs. TEE revealed aortic valve vegetations concerning for prosthetic valve endocarditis. https://www.instagram.com/p/CM7RAJ9h9yA/
Meme Nitro drip https://www.instagram.com/p/CM7UYi1JevS/
Yes Pathology Series Lung Pneumothorax Pathology A pneumothorax occurs when air becomes trapped between the visceral and parietal pleura of the lungs. A-lines are generally associated with normal lung, however they are also present in a pneumothorax. Since ultrasound waves don’t travel through air, the only signal that the transducer can pick up when imaging normal lungs is the pleural line. However, the transducer still assumes that objects exist beyond the pleural line. Although the ultrasound waves stops at the pleural line in normal aerated lungs, the transducer does not know this; therefore, the repeated signal from waves bouncing off the pleural line are plotted out as more distant structures. So A-lines are simply the repeating bright pleural line throughout the image since the lung is filled with air. Now considering the anatomy of a pneumothorax, since this is just air beyond the parietal pleura, you can see the problem….A pneumothorax will have A-lines just like a normal lung! So how do we differentiate the two? When the visceral and parietal pleura are in contact with one another, close observation of the pleural line will show movement back and forth as the visceral pleural slides along the parietal pleura. If a pneumothorax is present, even though you will still see normal A-lines, absence of lung sliding will indicate that the visceral and parietal pleura are not in contact with one another. Therefore, absent lung sliding is how a pneumothorax is diagnosed on ultrasound. Using the linear probe makes it much easier to see lung sliding. Keep in mind that since lack of lung sliding simply represents no movement of the visceral pleura there are other conditions where this may be the case. These include previous pleurodesis, bullous lesions, mainstem intubation, respiratory arrest, or adhesions in ARDS. The presence of a lung point which is an interface where you observe sliding and absence of lung sliding in the same image has a specificity approaching 100% for pneumothorax Artifacts like B-lines only exist when there is something besides air beyond the pleural line. So even though A-lines are present in pneumothorax, if you see B-lines then there cannot be a pneumothorax. https://www.instagram.com/p/CNF6gi3BdQl/
Specific Topic Cardiac Atrial fibrillation Afib with rapid ventricular response (RVR) Afib with RVR management Reduced cardiac output in Afib with RVR Afib fairly common thing to encounter in the emergency room. While ultrasound is less useful than EKG in recognizing Afib with RVR, it is a key component in the management. As always with every ED patient, your first few steps should be ABCs, then IV, O2, monitor. Once you have established access and placed the pt on oxygen if they are hypoxic, obtain a full set of vitals and place cardiac pads on the patient. First, lets talk about the stable patient. Key steps in management should focus on history and physical. Look at the ACEP flow sheet and exclusion criteria. Typically, consider causes of new onset afib such as volume depletion, holiday heart, hyperthyroid, infection, and PE. POCUS is a key component in my workup here, if the patient’s EF and wall motion appear relatively normal, normal RV size, and no significant signs of pulmonary edema or volume depletion then I feel much more comfortable discharging the patient. Most of the time patients are not exactly sure when their Afib started so rate control with something like PO metoprolol is generally my go to. I calculate their CHADS-VASC score and if ≥1 I start them on a DOAC prior to DC with cardiology follow up. If onset is known and the patient doesn’t have a complicated medical history or concerns for other underlying pathology on POCUS I generally perform electrical cardioversion and DC. Now the “unstable” patient is where most people start to have issues. The classic ACLS teaching says that patients that demonstrate any signs of instability (hypotension, AMS, shock, chest pain) they should receive immediate synchronized cardioversion. The decision to cardiovert should be slightly more nuanced in my opinion. If the patient is in Afib with a rate <150 it is unlikely that hypotension or AMS is a result of their arrythmia. In these cases the rate is likely compensatory and slowing them down may make things worse. Take a look at the IBCC flowsheet for management of Afib in critical illness. I think this is a great way to approach the pseudo stable patient in the ED. Don’t forget if hemodynamics allow the use of Diltiazem, the dose in most patients should be weight based at 0.25mg/kg for the first bolus. https://www.instagram.com/p/CNOWOl8BaQd/
Clinical Case Ocular Posterior vitreous detachment Propper gain for ocular ultrasound, use ocular motion to help evaluate the posterior segment Swirling in the posterior segment, PVD An 81 year old male presents with 24 hours of visual disturbances. He says there is no associated pain but feels like he started seeing floaters last night before going to bed. Denies trauma to the eye. Vision is baseline, external exam appears normal. You perform a POCUS. What is your diagnosis and how will you manage the patient? -- Answer Below – Diagnosis: Posterior Vitreous detachment (PVD) There are a few keys features in this image that suggest PVD rather than retinal detachment (RD). First if you look at the amount of gain in the images, despite a lot of gain, the tissue in the posterior segment is still fairly difficult to see. Compare this to prior cases of RDs I have posted. The retina is much brighter and more obvious. Additionally, observe the movement of this structure. As the eye moves the debris is fairly mobile. An RD is usually anchored to the back of the eye at a point near the optic nerve. Here you can see that is not the case and this debris almost swirls around completely as the eye moves. These are all features that suggest a PVD rather than an RD. That all being said, a retinal tear or small detachment is still a concern here. Generally speaking, an isolated PVD is benign and does not need to be treated. PVDs are very common in older adults with over 75% of people older than 65 developing them. However, without a detailed fundoscopic exam you may miss a small retinal detachment that needs to be dealt with urgently. The risk of RD is greatest in the first 6 weeks following PVD. For these reasons it is recommended that even if you suspect a PVD on POCUS, ophthalmology should be consulted to help with disposition of the patient. If the visual acuity is fairly normal and the patient is reliable, it is not unreasonable to have close outpatient ophthalmology follow up after discussing the case with them. A PVD can be partial or complete. As the vitreous separates from the retina this can tear retinal vessels and cause “floaters” or hemorrhage. In a complete PVD you may see a Weiss ring on POCUS. This represents the fibroglial tissue that is left free floating in the posterior segment. https://www.instagram.com/p/CNTO7pxhTu-/
Clinical Case Bowel Small bowel obstruction Hernia with SBO A 89 year old man with history of colon resection presents with 24 hours of worsening abdominal pain, nausea, and vomiting. He says at this point he cannot even keep water down. He notes a painful bump in his upper abdomen the size of a golf ball. You perform a POCUS of the area in question. What is your diagnosis and next step in management? -- Answer Below – Diagnosis: SBO 2/2 incisional hernia Management: After recognition of the hernia and SBO on ultrasound, manual reduction of the hernia was attempted at the bedside and was successful. CT scan was also performed and showed persistent dilated small bowel loops with no clear cause of obstruction (since it had already been resolved). The patient was admitted to the surgical service for serial abdominal exams and monitoring for resumption of normal bowel activity. As we have discussed before, ultrasound is good at detecting SBO (sensitivity 97.7%, specificity 92.7%), much better than abdominal plain films (sens 75%, spec 66%). *Below we have ultrasound features of SBO listed. Usually we get a CT to help identify the transition point, look for complete vs partial obstruction, identify less common etiologies (hernia or malignancy), and look for complications such as closed loop, ischemia, necrosis, or perforation. Here we were able to identify a hernia as the cause of the obstruction using POCUS and fix the problem at the bedside. Now what do you look for on POCUS for SBO? First make sure you are looking at small bowel. While large bowel will contain haustra, the Jejunum has plicae circulares, and ileum lacks haustra or plicae circulares. So once you see small bowel look for: 1. Dilated loops (>2.5cm in jejunum or >1.5 in ileum) 2. “to and fro” motion of the bowel contents 3. Lack of compressibility 4. Free fluid and bowel wall edema (>2mm thick) 5. Transition point Each element will increase you specificity, with a transition point being the most specific. https://www.instagram.com/p/CNYv2ApIcQ2/
Pathology Series Cardiac Pulmonary embolism Pathology Pulmonary embolism (PE) is a frequently discussed differential diagnosis in the ED. The test to diagnosis a PE (CTA) is expensive, time consuming, and exposes the patient to ionizing radiation, so we try to risk stratify patients in order to avoid unnecessary harm. First, you should first use a clinical decision aid like Well’s to evaluate the patient’s baseline risk. If the patient is low risk (score <2), then the risk of PE is ~4%. Low risk patients should be evaluated using PERC, if they PERC out (score 0) then their risk is <2% and no further workup for PE is indicated. If they are PERC positive (score >0) or moderate risk by Well’s (2-6 points ~20%), check a D-dimer. High sensitivity D-dimer is just that, highly sensitive, but not specific. Age corrected D-dimer is well accepted now and should be used. In patients >50, if using FEU, multiply age by 10 to calculate the cut off value. In younger patient’s use the lab cut off (500 for FEU). Patient’s with a positive dimer or those considered high risk by Well’s should undergo CTA. Now that all being said, this is an ultrasound post! How can ultrasound help us in the diagnosis of PE? One thing I will say off the bat is that IT SHOULD NOT be used as a screening test. You will only see signs of PE on ultrasound if there is right heart strain, therefore it is only helpful in risk stratification AFTER PE has been diagnosed. If you do however catch a clot in transit, that is a whole different story! Look at all the images depicting the ultrasound findings in PE. If you don’t remember how to use a likelihood ratio (LR) go back and look at my post on that. As you can see the LRs for most of these findings are not great, another reason why POCUS should not be used as a screening test for PE. Looking at McConnel’s sign, notice that it has very high specificity but isn’t very sensitive. Therefore, if you suspect a PE and perform an echo before you get a CTA, the presence of McConnel’s sign indicates that the patient likely has a massive PE. Look at the EM:RAP algorithm for low, intermediate, and high risk PE. Here you can see that echo findings are important for differentiating patients within the intermediate group. https://www.instagram.com/p/CNgW5pfhuxc/
Yes Specific Topic Cardiac Ejection fraction Infographics on every method of evaluating EF, EPSS, fractional shortening, Simpson's The mysterious ejection fraction (EF). Most experienced ultrasonographers just estimate the EF by looking at the heart in several views. However, when you’re getting started this isn’t really an option! So today I am going to show you a few methods have the ultrasound machine give you a numerical value for EF so you can start improving your visual estimation. One of the quickest, easiest, and least accurate ways to calculate ejection fraction is by measuring the E-point septal separation (EPSS). To acquire an EPSS measurement, get a good parasternal long axis view of and place the M-mode cursor over the tip of the mitral valve leaflet. Measure the distance from the MV leaflet to the intraventricular septum during the E-wave (first peak of 2 while the MV moves during diastole). Since this is an indirect measurement evaluating the size the LV relative to MV opening it isn’t good at giving an accurate EF. However, an EPSS >7mm is fairly sensitive for detecting an EF <50%. Note that aortic regurge as well as mitral stenosis can both give you a falsely elevated EPSS. Next, a more accurate method of calculating EF, is fractional shortening (FS). One of the main reasons FS is more accurate is that it is a direct LV measurement. First get a parasternal short axis view and place the M-Mode cursor straight through the LV chamber. Measure the maximum and minimum chamber sizes on the M-mode trace. If you are using the machines cardiac package you should get a FS % as well as an actual EF. FS > 28% is considered normal. Now the EF here is calculated using the Teichholz method, which assumes the LV is a fixed geometric shape. As you can imagine, any regional wall motion abnormality or asymmetric chamber dilation is not accounted for when using this method. Finally, the best and only ASE recommended method of calculating EF, biplane method of disks summation (modified Simpson’s rule). This is the most time-consuming method of measuring EF. Here you need at least an apical 4 chamber view (and for more accurate estimates both AP4 and AP2). Freeze the image in end systole and end diastole and trace the outline of the LV. The machine will calculate an EF using Simpson’s. https://www.instagram.com/p/CNoWDCMhSla/
Clinical Case Cardiac transesophageal (TEE) Cardiac Arrest Monitoring CPR quality using TEE Clots after prolonged CPR An 86 year old male arrives by ambulance complaining of weakness over the last 24 hours. In triage he is noted to be hypotensive and is rushed to the resuscitation bay. Several moments after being placed in the bed he appears unconscious and has no pulse. CPR is immediately initiated and he is hooked up to the monitor. A TEE is inserted and during the first pulse check there is agonal cardiac activity and the monitor shows PEA. Take a look at the first few TEE images here. The first shows the mid esophageal long axis view. You can see that the LA (closest to the probe) contains stagnant blood and the MV is not opening. There is minimal LV activity with a small amount of aortic valve opening. Given no large pericardial effusion as a cause of arrest, our focused moved to evaluation of the RV and pulmonary artery. The omniplane was adjusted to visualize the RV inflow outflow window. While the RA does appear enlarged there is no clear thrombus identified. Mechanical CPR was resumed. The next shot shows the main and right pulmonary arteries also without clear thrombus. There was some difficulty optimizing location of mechanical CPR to achieve optimal LV output, as can be seen in the TEE images. CPR continued for approximately 45 minutes with several intermittent episodes of ROSC. After one episode of briefly sustained ROSC, large clots began to appear in the LA and LV. Given the overall duration of arrest without an identified cause the decision was made to terminate resuscitation efforts. Although this is a heartbreaking outcome, this case highlights a potentially devastating and underrecognized complication of sustained cardiac arrest that would inevitably lead to poor neurologic outcomes independent of sustained brain hypoxia. https://www.instagram.com/p/CNx3cojhnZc/
Basics Series Cardiac transesophageal (TEE) TEE Basics - Part 1 Basics Transesophageal echocardiography (TEE) is becoming among acute care providers across the country. TEE provides outstanding views of the heart and can be used during CPR to help guide a resuscitation. For anyone who has tried to perform transthoracic (TTE) echo during CPR, you know that it is almost impossible to get much quality information. When it comes to first learning TEE the biggest hurdle is understanding the omniplane. Adjusting the rotation of the US beam in TTE only requires you to turn your hand, since the TEE is within the patient’s esophagus, obviously this isn’t possible. That is where the omniplane comes in. Notice the half circle in the at the top of TEE images. This indicates the current ultrasound beam angle (omniplane). To obtain different axis views while using the TEE, the operator adjusts the omniplane, which rotates the US beam within the tip of the probe, anywhere between 0-180 degrees. By combining this with the other planes of motion (insertion depth, L/R rotation, flexion) you can visualize the heart along any potential axis. Care must be taken when inserting the TEE to avoid any oropharyngeal trauma. Always make sure the flexion wheel on the TEE is unlocked so that the tip is not rigid. In the ED we only perform TEE on intubated patients so using a laryngoscope can help facilitate direct insertion into the esophagus. If inserting without direct visualization make sure to only apply gentle forward pressure and stop if you encounter any resistance. After insertion, manipulating the TEE probe is easy once you get the hang of it. I am right handed and prefer to hold the transducer in my left hand with my index and middle fingers on the omniplane buttons. I hold the shaft of the TEE overhand with my right hand so that I can easily rotate it R/L. Rotation and insertion depth of the probe are the main ways you identify the structure you are trying to image. I also use this hand to advance or withdraw the probe. Once I have the centered the structure in question, I adjust my omniplane to achieve the desired view. https://www.instagram.com/p/CN28bMMh3UG/
Basics Series Cardiac transesophageal (TEE) TEE Basics - Part 2 Basics Now for part two I am going to focus on imaging during the TEE and what I think a good “resus” TEE protocol should include. University of Toronto has an amazing FREE TEE simulation app that is super helpful, and I have used some screenshots from it here. After insertion advance the probe to a depth of ~30cm while keeping an eye on the ultrasound screen. As you advance, the first view you will likely see is either the mid esophageal 4 or 5 chamber. Adjust the rotation, insertion depth, and omniplane until you achieve a good ME4 view. Here you can evaluate the LV function, LV:RV size, look for a pericardial effusion, and identify any valve or wall motion abnormalities. Orienting yourself can be difficult. I use a memory tool to help me with this. With the patient supine, if I am at the head of the bead, my right hand facing the patient represents the omniplane at 0 degrees. Now the omniplane rotates clockwise, so at 90 degrees my pinky is facing the patient’s feet and my thumb is facing me. Now, your palm represents the image seen on the ultrasound screen at any omniplane, so at 90 degrees (pinky facing the feet) inferior structures are on screen left. If you ever get lost, just remind yourself that LA is almost always the closest structure to the probe and then you can connect the dots from there. Now that you understand how to interpret omniplane rotation, lets get the next view. Once you have a nice ME4, hold that position and rotate the omniplane to ~130 degrees. Now you should see the LV connecting to the LVOT and the aortic valve opening and closing. This is the best view to assess quality of CPR in an arrest, here you should see good LV compression with corresponding aortic valve opening. If the aortic valve is not opening, compressions should be modified. Next, adjust the omniplane to 0 degrees and advance the probe until you see the transgastric mid short axis view. If there is nothing else I am assessing during the TEE I just hang the probe on an IV pole and leave the screen on this view. Now everyone in the room has a constant assessment of LV function throughout the remainder of the resus. Other views included here for more specific evaluations. https://www.instagram.com/p/CN3Lbi5h3rB/
Clinical Case IVC Vascular IVC Thrombosis Management of IVC thrombus An 89 year old female presents with past medical history of hypertension. She presents with acute shortness of breath, tachycardia, epigastric and chest pain. You perform a bedside echo which is unremarkable. You next focus on the abdomen to evaluate her epigastric pain and find the above images. What is your diagnosis and next steps in management? -- Answer Below – Diagnosis: IVC thrombosis Management: From and ED perspective, IVC thrombus management should focus on ruling out life threatening complications as well as preventing further propagation of the thrombus. Since this patient had associated SOB, tachycardia, and CP, it was assumed that she had a PE in addition to her IVC thrombus. A CTA revealed extensive BL PEs and confirmed our echo findings that there were no signs of RV strain. The patient was admitted on a heparin gtt with a PERT (PE response team) consult. The PERT includes interventional radiology who elected to take the patient for suction thrombectomy where they removed 5-6 cm of thrombus from the IVC. Further workup has not revealed the source of her extensive clotting. Hematology workup included Factor V Leiden, prothrombin genotyping, antiphospholipid antibody, and malignancy screening, which were all negative. The patient was continued on a heparin gtt while inpatient and transitioned to Eliquis for discharge. https://www.instagram.com/p/CN8QvFbBQ9g/
Echo Cases Cardiac Aortic valve Severe aortic stenosis Assessment of aortic stenosis ASE aortic stenosis tables Echo Cases Echo case #2 – An 80 year old male presents with worsening DOE x 1 week and acute syncope today that prompted his visit to the ED. He admits to history of hypertension. -- Answer Below – Diagnosis: Likely severe aortic stenosis (AS) AS is the most common primary heart valve disease. Currently the ASE recommends combining all Doppler and 2D data as well as clinical presentation in the evaluation of AS. Loading conditions influence velocity and pressure gradients; therefore, these parameters vary depending on intercurrent illness of patients with low vs high cardiac output. The most common causes of AS is calcific stenosis. Tricuspid valve AS predominates in elderly patients >75, rheumatic disease is uncommon in North America and Europe but still prevalent worldwide. The severity of valve calcification can be graded semi-quantitatively, as mild (few areas of dense echogenicity with little shadowing), moderate (multiple), severe (extensive). The degree of valve calcification is a predictor of clinical outcome including heart failure, need for valve replacement, and death. The primary hemodynamic parameters recommended by the ASE are: peak jet velocity, mean transvalvular pressure gradient, and AV area. Peak jet velocity is measured by placing continuous wave doppler in the LVOT from an apical 4 view. The CW doppler must be parallel to the outflow jet. >4 m/s is considered severe. In mild obstruction, the peak is in early systole with a triangular shape, a more rounded curve and mid systolic peak suggests severe stenosis. Our AV peak velocity is 2.6 m/s and the curve appears normal. AV area is calculated using the continuity equation. This assumes all blood exiting the LVOT enters the AV, therefore, standard calculation requires 3 measurements: AV jet velocity (CW), LVOT diameter, and LVOT velocity (PW). You can see that despite a normal peak velocity, the continuity equation yields and AVA of 0.7 cm2 suggesting severe disease. Personally I like the dimensionless index given how easy it is. This uses the ratio of LVOT to AV VTIs. DI approaches 1 in normal AVs and measurements less than 0.25 are considered severe AS. Here we see the DI in our patient is 0.2 https://www.instagram.com/p/CN8iUsKheL7/
Yes Pathology Series Gallbladder Cholecystitis Pathology While acute calculous cholecystitis (ACC) is a fairly common diagnosis in the acute care setting, diagnosing it remains somewhat complicated. Most would agree that ultrasound is the preferred first line diagnostic modality. It has a sensitivity of ~81% and specificity of ~83% depending on what literature you look at. There are 5 ultrasonographic findings of acute cholecystitis: 1. Non-mobile stone in neck (SIN sign) 2. GB wall thickening > 4mm 3. Hydropic size >5x10cm 4. Pericholecystic fluid 5. Positive sonographic murphy’s sign Despite ultrasound being fairly sensitive and specific for cholecystitis, most surgical societies agree that no single feature is sufficient to take the patient to the OR. Overall clinical picture is necessary to diagnose ACC. In 2020 WSES recommended that US be the first line test to diagnosis ACC. They noted that if US and clinical signs are equivocal then further imaging with HIDA or MRI should be pursued. They note that diagnostic accuracy of CT is poor for ACC. Additionally, WSES states that the Tokyo Guidelines perform poorly outside of the original derivation study, however, the clinical features of the TG18 score above give you an idea of the other diagnostic considerations. Overall, while the literature is poor on the exact sensitivity and specificity of the various ultrasound signs of cholecystitis, there is agreement that each sign increases your specificity. Additionally, the presence of gallstones (not SIN) along with any one US finding of cholecystitis showed a sensitivity of 87% and a specificity of 82% for ACC. RUQ pain and any findings of ACC on POCUS should prompt a surgical consult. While ACC is primarily an inflammatory rather than infectious process, antibiotics are generally initiated since prolonged obstruction can lead to infection. Just know that overall, the diagnosis of ACC needs both clinical and imaging components, so don’t be upset if your surgical colleagues request further workup prior to admission. https://www.instagram.com/p/CN-kjGuBQGq/
Clinical Case Obstetrics (OB) Gynecology (GYN) Incomplete abortion Know your anatomy Products passing through the cervix A 27Y G4P2 female presented to an OSH with heavy vaginal bleeding and hypotension. At OSH she had a transvaginal ultrasound done that was concerning for ruptured ectopic pregnancy. On an ED to ED phone call the outside Dr. states that radiology read the ultrasound as “a previously present (2 weeks ago) IUP is now absent and there is a 2×4 cm complex mass adjacent to the left adnexa concerning for heterotopic pregnancy.” The OSH doc notes that there is no mention of free fluid on the radiology read. A unit of PRBCs is started at the OSH and she is transferred to your ED. Upon arrival to your ED she is hypotensive 80/50 with 1 U PRBCs infusing. She doesn’t know when her LMP was definitively, but her best guess puts her around 14 weeks. She is complaining of abdominal pain and continued vaginal bleeding. You obtain a bedside ultrasound. What do you see? -- Answer Below – The images show an incomplete abortion with tissue within an open cervical canal. This is a sad case but shows the importance of identifying all relevant anatomy when performing an ultrasound. The advantage we have in POCUS, vs radiology reading ultrasounds performed by someone else, is we are physically at the bedside and can immediately clarify any confusing parts of the image. In the first few clips it is not quite clear how all the structures are connected. The “complex mass” could easily be mistaken for something outside of the uterus. As we change to the sagittal orientation in the last few images you can tell that there is tissue within the uterus, traversing the cervix (which is open) and remaining in the vagina. There is only a small amount of pelvic free fluid, making a ruptured ectopic unlikely as the cause of her hypotension. Our ED team discussed the findings with OB, given the fact that the patient remained unstable she was taken for D&C. However, with the confusion from the outside read the decision was made to also laparoscopically rule out any ectopic pregnancy. The team found normal bilateral adnexal anatomy and confirmed incomplete abortion as the diagnosis. https://www.instagram.com/p/COIRs8Vh-aV/
Clinical Case Bowel Small bowel obstruction recognition of jejunum Infographic with plica circularis in the jejunum, transition point A 64 year old female with history of large ventral hernia presents with 24 hours of worsening abdominal pain, nausea, and vomiting. She says at this point he cannot even keep water down. She says that there is a sensation like she is wearing a tight band around her hernia. You perform a POCUS of the area in question. What is your diagnosis and next step in management? -- Answer Below – Diagnosis: SBO within a large hernia Management: Unlike a previous case where I reduced the hernia at bedside, here after recognition of the SBO on ultrasound, I did not attempt reduction given the significant size of the hernia. CT scan was performed and showed dilated bowel within the hernia confirming a SBO. The patient was admitted to the surgical service, non-operative management was attempted given how large the hernia was, however this failed and pt was ultimately taken to the OR. As we have discussed before, ultrasound is good at detecting SBO (sensitivity 97.7%, specificity 92.7%), much better than abdominal plain films (sens 75%, spec 66%). *Below we have ultrasound features of SBO listed. Usually, we get a CT to help identify the transition point, look for complete vs partial obstruction, identify less common etiologies (hernia or malignancy), and look for complications such as closed loop, ischemia, necrosis, or perforation. Here we were able to identify a hernia as the cause of the obstruction using POCUS. Now what do you look for on POCUS for SBO? First make sure you are looking at small bowel. While large bowel will contain haustra, the Jejunum has plicae circulares, and ileum lacks haustra or plicae circulares. So once you see small bowel look for: 1. Dilated loops (>2.5cm in jejunum or >1.5 in ileum) 2. “to and fro” motion of the bowel contents 3. Lack of compressibility 4. Free fluid and bowel wall edema (>2mm thick) 5. Transition point Each element will increase your specificity, with a transition point being the most specific. https://www.instagram.com/p/COatqEhhF_w/
Clinical Case Testicular Epididymo-Orchitis Color flow images in orchitis Testicular mass, reactive hydrocele A 48 year old male presents with 5 days of left testicular pain. He saw his PCP and was referred to the ED to rule out testicular torsion. The patient says that his pain has been progressively worse and now severe even with light touch to the left testicle. Admits to mild swelling and denies any pain with urination. You perform a POCUS, what is your diagnosis and next step in management? -- Answer Below – Diagnosis: Epididymo-Orchitis Management: Most cases are treated outpatient with oral antibiotics and NSAIDS. If the patient appears ill or is septic they should be admitted. Empiric abx are generally given based off the patient’s age. Classically, patients <35 years old are considered at risk for STIs, while patients >35 are not. I prefer to just ask the patient about their sexual practices. High risk features such as multiple partners or non-monogamous relationship without use of protection, as well as anyone practicing anal intercourse should be considered at risk for STIs regardless of age. At risk for STI = coverage for gonorrhea and chlamydia. The CDC just recently updated their recommendations on empiric treatment to Ceftriaxone 500 mg IM (1G IM if weight >150 kg) and doxycycline 100 mg BID x 10 days. If also practicing anal intercourse the CDC recommends using levofloxacin 500 mg BID x 10 days rather than doxy for better coverage of enteric organisms. Low risk for STI = Coverage should focus on enteric pathogens alone so Levofloxacin 500 mg BID x 10 days. Diagnosis should be made by a combination of clinical and ultrasonographic features. UA and culture should be sent as well as NAAT for STIs. Clinically epididymitis presents as localized pain with tenderness in the affected testicle. In cases of epididymo-orchitis, ultrasound diffuse hypervascularity (tiger striping) of the testicle as seen here. Other features include hypoechogenicity, swelling, scrotal wall thickening, and a reactive hydrocele. Also, of note there was an incidental finding of a lobulated area in the lower pole of the right testicle. This was possibly from a prior injury, however we recommended that the patient have a 3 month follow up ultrasound to reevaluate this area. https://www.instagram.com/p/COeDstThE-w/
Yes Basics Series Bowel Bowel Basics Ultrasound the bowel is a very useful and IMO underutilized POCUS application. Today I am going to cover some of the basics to help get you started. Individual pathologies will be covered in other posts, this one will focus on normal anatomy and scanning technique. In general I start with the curvilinear probe. Make sure your depth is appropriate ~10 cm or less and be generous with the gel. Normal bowel compresses very easily so use as little pressure as possible in order to identify the layers of the bowel wall and then you can use graded compression when looking for certain pathologies. Depending on what you are looking for your scanning process will vary. For SBO, I use the “lawnmower” technique. For appendicitis I identify cecum, find the terminal ileum and then look for the appendix adjacent to that. For diverticulitis I start looking right at the location of maximum tenderness. Starting out with bowel ultrasound can be difficult. I think the first thing you should always do when looking for bowel is identifying the peritoneum. Once you see where that is you know that anything you see below that line is intrabdominal. Take a look at the clip where I labeled rectus and peritoneum, see if you can follow the peritoneum up and identify what structures are intrabdominal and which are not. So obviously you have large bowel and small bowel. You can identify large bowel by the undulations created by the haustra. Small bowel has plicae circulares starting from the 2nd part of the duodenum, thickest in the jejunum, and usually not seen in the ileum on ultrasound. The plicae are really only visible with the small bowel is dilated. Check out a few of the pathologic clips at the end and see what you can identify. https://www.instagram.com/p/COtTD1wBp1F/
Meme Enhance https://www.instagram.com/p/COvwncuheOG/
Clinical Case Cardiac EKG ST elevation myocardial infarction (STEMI) If there is concern for infarct look for regional WMAs Infographic for wall motion abnormalities A 60 year old male comes in with severe upper back pain that woke him from sleep. He says that he has never had pain like this before. He admits to radiation of the pain to his chest and mild SOB. He says that he is healthy and runs marathons all the time. Take a look at his EKGs and the echo. What is your diagnosis and dispo? -- Answer Below – Diagnosis: Acute myocardial infarction with anterior lateral and apical wall motion abnormalities Discussion: The EKGs here are very interesting. The EMS EKG shows hyperacute T waves in V2 and V3 with STE that doesn’t quite meet STEMI criteria. Inferiorly in 2, 3, and aVF there is TW depression and inversion. This EKG was flagged as high alert after EMS transmitted it to our ED and patient was brought directly to a high acuity bed upon arrival. Repeat EKG showed similar changes but with less STE in the anterior leads and some new lateral depressions. Given the patients story of severe back pain that woke him from sleep dissection was high on the list of differentials. An immediate POC echo was performed that showed a normal aortic root and arch. The cath lab was activated given the normal arch and dynamic EKG changes. The echo also revealed a an apical and anterior lateral WMA that sealed the deal for cardiology to take the patient to cath. Angio revealed 100% LAD occlusion, consistent with echo findings. WMAs can be very difficult to pick up on. The best way that I have found for myself is to observe for thickening and thinning of the myocardium. As the heart contracts, especially on the parasternal short, you will see the muscle thicken and thin in areas of normal motion. In the areas where the heart is not contracting well the tissue will appear to stay the same thickness. Take a look at the image with the vascular territories labeled and then watch the clip without the labels. You can see that in the anterior lateral LAD territory there is no real thickening or thinning of the myocardium compared to the rest of the heart. You can also see this in the apex on the apical 4. https://www.instagram.com/p/CO_CWWLBOzv/
Yes Pathology Series Testicular Testicular Torsion Pathology When it comes to testicular ultrasound in the emergency room there is one diagnosis we are really worried about and that is torsion. Tumors, infections, fluid collections, these can all be dealt with less urgently. Generally all of these issues can wait to undergo a complete sonographic assessment by radiology or be transferred to a center that performs scrotal ultrasound. However, if you can diagnose torsion on POCUS, this allows you to obtain early surgical consultation and likely save the patient’s testicle. Therefore, I think every acute care provider should understand how to use POCUS as a RULE IN for testicular torsion. Use the linear probe and place it at the bottom of the scrotum with the indicator facing the patient’s right side. This should give you the “buddy” view where you can see both testicles side by side. In this view you are going to grossly compare the echotexture of each testicle, and look for obvious hydrocele. Next, place a single color flow box over both testicles to compare flow. Increased flow can be seen in inflammatory conditions or post detorsion, while absent flow is concerning for acute torsion. Now move on to individually imaging the testicles. First, go to the normal one. The testicle should appear homogenous through and have minimal to no surrounding fluid. After this place a color flow box over the normal testicle and adjust the gain of the until there is apparent flow with minimal noise. This gain setting will be left when imaging the other side. Now repeat this on the testicle of concern. Specifically, when looking for torsion, if you place the properly calibrated color flow box over the testicle in question and see no flow, this should be considered a positive exam for torsion. Some secondary signs of torsion to keep an eye out for would be elevated RI >~0.7, reactive hydrocele, and thickening of scrotal skin. Keep in mind that a recently detorsed testicle can appear hyperemic mimicking orchitis. At the end of the day this should be used as a RULE IN. If there is flow in both testicles, send the patient off for formal radiology ultrasound. If flow is absent in one testicle, call urology. https://www.instagram.com/p/CPD4A-QBMTL/
Clinical Case Lower Extremity Musculoskeletal Septic Joint Management of a septic joint TEE with aortic valve vegetation A 68 year old male presented to the ED with a 1 week history of right knee and right should pain. He says that he was seen at urgent care a few days ago and prescribed steroids. He notes that since then his blood sugars have been very high and his pain has gotten worse. His daughters made him come to the hospital today because he can no longer walk because of the knee pain. He denies fevers but admits to chills. He denies any trauma. He says that since this morning he has been very nauseated. The joints in question are mildly warm and extremely painful with passive ROM. There is no overlying erythema but there is obvious swelling of the knee. You perform a POCUS evaluation of each joint. What is your top differential and next step in management? -- Answer Below – Top Differntial: Septic joint Management: Diagnostic arthrocentesis, broad spectrum antibiotics and admission. The POCUS shows a very large effusion in the right knee and a moderate effusion in the right shoulder. Given the patients exam with a history of chills and severe nausea this picture is very concerning for septic arthritis. Polyarthralgia is less common in septic arthritis than rheumatic disease, therefore fluid was also sent for crystal analysis. Arthrocentesis showed severely elevated WBC with gram positive cocci in pairs and clusters identified on gram stain. Additionally the original read from microbio also stated that there appeared to be gram negative intracellular organisms so disseminated gonococcal infection was also entertained. The patient was started on Vancomycin and Rocephin while in the ED. ID and orthopedics were consulted. Further hx from the patient revealed no concerning features for STIs. Blood cultures resulted in MRSA so hematogenous spread was considered the most likely source. Patient underwent a TEE evaluation that showed a small vegetation vs calcification on the aortic valve. He underwent arthrotomy with orthopedics. Subsequent blood cultures had no further growth after the patient was transitioned to Daptomycin. He was discharged home with 6 weeks of PICC IV antibiotics for presumed endocarditis along with outpatient thoracic surgery evaluation. https://www.instagram.com/p/CPLkd95Bn4b/
Yes Basics Series Musculoskeletal Musculoskeletal Basics Basics Musculoskeletal ultrasound is extremely useful in the acute care environment. Since Xray and CT are not great at diagnosing tendon/ligament injuries, ultrasound is a good alternative when MRI is unavailable or impractical. However, many ER docs are hesitant to use ultrasound for MSK issues due to lack of training. MSK isn’t something that is frequently focused on in residency and therefore seems like a niche and difficult topic to many. Well, I am here to tell you that MSK ultrasound is actually fairly easy!! When using POCUS to diagnose musculoskeletal injuries, there are a few key things to remember. 1. Before you start your MSK scans look up some diagrams of the anatomy in questions. If you are looking for a patellar tendon rupture, make sure you understand what should be connected where before starting. If you are trying to find a bursitis make sure you know where the bursa should be located before you go to scan. 2. Once you are in the room with the patient, always find something you can identify and trace the anatomy back from there. In the knee example above, I think the patella is the best structure for this. You can feel where the patella is and then place your probe right over it. I usually start with the curvilinear probe for large joints and then switch to the linear probe. 3. Compare your findings to the opposite side. Some people start on the injured side, I usually like to start on the normal side and identify what the patient’s normal anatomy looks like. After this I click the “dual” button and find the structure in questions on the abnormal side and see if there is a difference. Take a look at the Achilles tendon rupture image above as an example. 4. Finally, just remember to think about the normal principles of ultrasound. Bone will appear bright white and have shadowing behind it, fluid and effusions will appear hypoechoic. If you always remember to fall back on these key principles, you’ll be diagnosing tendon ruptures and muscle tears in no time! https://www.instagram.com/p/CPUKBYth7wu/
Clinical Case Bowel Appendicitis Findings of acute appendicitis, identify the terminal ileum Infographic with outline of cecum and appendix A 22 year old male presents to the ED with acute onset lower abdominal pain. He denies past medical history. He says the pain is worse with movement and seems to localize to the RLQ. He denies vomiting but admits to mild nausea. You perform a POCUS of the RLQ, what do you see and what is your next step in management? --Answer Below— Diagnosis: Appendicitis Management: Surgical consult is the first step. If there is high clinical suspicion and the patient appears ill it may be appropriate to involve surgery prior to imaging. If there are features of complicated appendicitis (perforation, abscess, or phlegmon) on imaging, or if they are systemically ill, antibiotics should be started immediately (Zosyn). If the are no signs of complicated appendicitis, antibiotics initiation should be coordinated with the surgery team (generally Cefoxitin or Cefotetan). The reason you should wait and talk to surgery is because if they are going to the OR the same day, the recommendation is to give the dose of prophylactic antibiotics within 60 minutes of first incision. Beyond antibiotics you should focus on pain control. Make sure to keep the patient NPO for the OR. Remember, normal lab work DOES NOT rule out appendicitis. On the images you can see a blind ending pouch coming off of the cecum, this represents the appendix. Don’t get confused by the terminal ileum which is in the same area and looks similar. You have to confirm that you are identifying a blind ending structure. Notice in the short axis clip you can see compression over the appendix. What are the signs of appendicitis on ultrasound? Aperistaltic, non-compressible, dilated appendix (>6mm) Hyperechoic appendicolith with posterior shadowing Peri-appendiceal fluid collections Wall thickening (>3mm), mural hyperemia on color flow increases specificity https://www.instagram.com/p/CPjC042hd3f/
Clinical Case Obstetrics (OB) Hemorrhagic Shock Ruptured ectopic pregnancy trauma management Positive fast A 27 year old female presents to you ED complaining of sudden onset severe lower abdominal pain. She denies previous medical history. On exam the patient is very uncomfortable. BP is 97/40 and HR is 115. You perform a POCUS. What is your diagnosis and next steps in management? -- Answer Below – Diagnosis: Ruptured ectopic pregnancy Management: Immediate management must focus on resuscitation and prompt surgical consultation. Remember that resuscitation in a ruptured ectopic pregnancy should be treated like a trauma. This is a patient with hemorrhagic shock. Younger patients compensate for acute blood loss fairly well and a borderline hypotensive and tachycardic patient should scare you! The oxygen carrying capacity of crystaloid is ZERO, therefore in hemorrhagic shock blood products should be your ONLY fluid of choice. As you can imagine, when the issue is losing RBCs, which can deliver oxygen to the tissue, further dilution of the concentration of RBCs with salt water does not make much sense. The 10th edition of ATLS still recommends up to 1L of warmed crystalloid. However, the literature suggests that even this is likely harmful. Therefore, it is recommended to allow for permissive hypotension when blood products are not available. There is some debate about when you should attempt crystalloid if PRBCs are unavailable. Fortunately, I work at a large center where this is not the case, however, if you are a pre-hospital provider or work at a small center without blood this is something you might face. I think that if the patient has normal mentation then hypotension should be allowed to avoid crystalloids. The above ultrasound images show free fluid in the RUQ. In this patient, free fluid on a FAST with or without a visualized ectopic should prompt and immediate consultation to OBGYN while a pregnancy test is pending. Here we also see that when looking at the pelvis there is more free fluid and a large area of hemorrhage with what appears to be clot. The diagnosis was immediately recognized by the ED provider and the patient was taken to the OR. She recovered without issues after surgery. https://www.instagram.com/p/CPn8S7TBHfF/
Echo Cases Cardiac RV failure RV failure rv spiral of death dilated RV, D sign Echo Cases A 72 yea old male presents with increasing SOB. Upon arrival you note his oxygen saturation to be 85%. He is unable to tell you his medical history but says that he usually wears oxygen at home. You perform a POCUS. What are your major differentials and what are your next steps in management? -- Answer Below – Major differentials: PE, pulmonary artery hypertension, RV infarct The above echo shows a moderately reduced ejection fraction ~40% along with several signs of RV failure and pressure overload. PS long shows a dilated RV with a pacer wire. The LVOT and aortic valve appear normal. LV function is reduced and there is a prosthetic mitral valve (note the tube shaped structure). The LA appears enlarged. In PS short there is flattening of the intraventricular septum with bowing into the LV. Under normal conditions LV pressure > RV pressure so the septum should bow into the RV. Also note that the RV appears large compared to the LV in this view. Normal ratio in the AP4 is 0.6 : 1, however in cases of severe dilation you can even recognize the abnormal ration on parasternal short. AP4 shows severely reduced global RV function. When dealing with acute RV failure recognized on POCUS pulmonary embolism, myocardial ischemia and pulmonary hypertension should be your top differentials. The degree of pressure overload and RV dilation here is atypical for acute MI and the EKG did not show IWMI so that pushed us towards PE. CTA revealed no PE and the patients’ spouse arrived and confirmed a diagnosis of pulmonary artery hypertension. Bowing of the RV into the LV leads to decreased CO. Increased RV wall tension leads RV ischemia. Hypoxia and hypercapnia both increase RV pressure via pulmonary vasoconstriction. As you can see, RV failure is the “perfect storm” for peri-intubation arrest. Start pressors and inotropes early in these patients. Response to fluid boluses should be monitored closely as this can improve or worsen the situation. If PE or MI are identified fix those problems ASAP. If you are dealing with isolated RV failure then reduction of RV afterload with medications such as inhaled nitric oxide can be considered after correcting hypoxia and hypercapnia. https://www.instagram.com/p/CP3kvcVhhKz/
Clinical Cases Cardiac transesophageal (TEE) Learning from mistakes Normal TEE always consider differentials, remain humble, learn from your mistakes artifact that appeared like a dissection flap A 78 Y M is brought in for cardiac arrest. Per family patient was feeling short of breath so they went took him to see his Dr. While in the waiting room he collapsed and was found to be pulseless. CPR was initiated and he was brought to our ED. ROSC was achieved shortly after arrival and a TEE was performed to identify any reversible causes. What do you see? -- Answer Below – This is a learning case for me! This TEE was looking normal until I went to evaluate the patient’s aorta. There appears to be a flap in the ascending aorta that had me concerned for dissection. Now the story here does not make much sense for dissection. The patient had SOB, collapsed, and then achieved ROSC. There are really 2 main ways a dissection can cause sudden pulseless arrest. A type A dissection that communicates with the pericardium and causes pericardial tamponade. This is obviously not the case here as we can see there is no large effusion. A dissection that involves a coronary artery and causes an arrythmia such as V-tach or V-fib. Now, another possibility is that the patient never actually had cardiac arrest and instead the dissection extended into the carotid artery(s) and caused cerebral anoxia causing him to faint. Since he was obese it was difficult to palpate a pulse, so I thought this was a consideration. However I evaluated both carotid arteries and did not see a flap. Given that the clinical picture didn’t fit well I wasn’t sure that what I was seeing was real. However, I looked in a few different angles and convinced myself that something was there! The patient went for CTA and there was no dissection. He was taken to the ICU and after several days of workup they concluded that he had cardiac arrest due to hypoxia from COPD. I sent this case to @thedadesthesiologist and his immediate response to my clips was looks like artifact. I asked him how he could tell, and he just said from doing hundreds of TEEs it is just something he sees frequently. I just wanted to post this experience to remind everyone to remain humble. I think one of the best ways to keep our patients safe is to keep asking ourselves, “How could I be wrong about this diagnosis or management." https://www.instagram.com/p/CQYy4IXBizB/
Clinical Cases Cardiac Pulmonary embolism (PE) Pulmonary embolism (PE) Massive PE management McConnell sign, D sign, RV dilation, TEE A 62 Y F with extensive psych history who is a frequent flyer to your ED arrives complaining of CP and SOB. She is tachycardic to the 140s, tachypneic in the high 20s, with a normal blood pressure, normal sat, and clear lung sounds. You perform a POCUS. What do you see and what are your next steps in management? -- Answer Below – Thankful to say that one of our superstar senior residents picked up this patient when she came in. Many people would not have even been worried about her given the fact that she is a frequent flyer for psych issues. However, this resident realized the importance of listening to the patient and looking at the entire picture. With those complaints and such abnormal vitals, he immediately put a probe on the patient’s chest. His POCUS shows RV dilation and AP4 shows a McConnell’s sign. He called CT and asked them to clear the table for a CT PE study to be done. He signed paperwork to bypass labs and the patient was scanned just a few minutes after being roomed. Unfortunately, just as she was being moved from the CT table to her ED stretcher the patient lost pulses. CPR was started immediately and TPA given. In crashing patients give 20 mg IV alteplase as a push and the remaining dose over 2 hours. The goal of lytic therapy here isn’t to completely resolve the clot, it is to help normalize PA pressures and prevent sudden cardiac death. The patient achieved ROSC shortly after. The ED TEE team was called for a peri arrest echo. A few of those images are shown as well. You can see severe RA and RV enlargement, however like we have discussed before this can be normal post arrest. This degree of enlargement is very concerning though given a relatively short down time. Unfortunately, no clot in transit or MPA clots could be seen on the TEE, but thanks to our residents’ quick action the CT had already been completed and showed extensive bilateral pulmonary emboli. The patient was rushed to the IR suite where mechanical thrombectomy was performed. Patient was taken to the ICU and made a full recovery. Needless to say this resident quick thinking and superior POCUS skills saved this patients life! https://www.instagram.com/p/CQcQjM8hcj5/
Meme Consultant on call "Thank you for this interesting consult." https://www.instagram.com/p/CQeWeAXhDpj/
Clinical Cases Lower Extremity Musculoskeletal Distal medial gastrocnemius head tear Evaluate both sides, msk injury management Calf muscular rupture A 28 year old male was participating in a Tuff Mudder race when he felt a “pop” in his right ankle. He says it felt like someone hit him in the leg. He was able to walk about a mile on it after the injury but then the pain became unbearable and he called for the medical team to pick him up. He arrives to your ED for evaluation. You perform a POCUS. What is your diagnosis and management? -- Answer Below – Diagnosis: Distal medial gastrocnemius head tear Management: Walking boot, rest, NSAIDs, orthopedic follow up After hearing this patients history from the PA I felt that he was likely going to have a partial or complete Achilles tendon rupture. However on my exam he had fairly normal strength of the right calf despite pain. You can see that his Achilles tendon is clearly intact, both the left and the right side images appear the same with the tendon inserting on the calcaneus with no interruption of the fibers. After seeing this I asked the patient to indicate the spot where he was experiencing the most pain. He pointed to a spot on his medial mid-calf. I placed the linear probe directly over this area and noticed that the muscle fibers appeared to be somewhat disrupted here. When comparing to the other side it is clear that there is a difference in tissue continuity along the musculotendinous junction of the medial head of the gastrocnemius. Most injuries to the calf muscle are treated with immobilization and physical therapy. When there is a complete rupture of the Achilles tendon I prefer to splint patients in an “equinus” position with the foot pointed slightly down, this allows for some approximation of the ruptured tendon while awaiting orthopedics evaluation. In this case I simply placed the patient in a walking boot and provided him with crutches recommending NWB of that extremity until he was evaluated by orthopedics. https://www.instagram.com/p/CQwXJ-JhBql/
Yes Pathology Series Pneumonia Pneumonia management of pneumonia Pneumonia flow sheet, air bronchograms, subpleural consolidations, IDSA guidelines Pathology Series Pneumonia (PNA)….It seems like such a simple topic, but unfortunately it isn’t. First we need to decide on what type, is this community-acquired (CAP)? Or hospital-acquired?.....Aspiration, atypical, bronchitis, viral. As you can see it quickly becomes a lot more complicated than just PNA. Ignoring all this confusion, how do we even diagnose it!? Well, I am here to tell you that ultrasound is key! The most accepted “definition” of bacterial PNA is a constellation of fever, respiratory symptoms, abnormal lung sounds, AND/or infiltrate(s) on CXR. However, up to 33% of older patient’s don’t have a fever, and >50% actually present with just confusion! Further, study’s looking at CXR vs CT show that around 30% of patients with PNA seen on CT have a normal CXR. That means that about a third of elderly patients you see with a mild cough, no fever, and a normal CXR might have bacterial PNA! Well, we can’t just CT everyone in this group and we shouldn’t just blindly prescribe antibiotics to everyone, so what are we to do!? Thankfully, ultrasound has shown better sensitivity than CXR for detecting PNA across multiple studies. When you have a moderate suspicion for PNA, especially in an elderly patient, with a negative CXR. My next step is to decide if they are going to be admitted. I use a score such as CURB-65 or PSI to answer this question. For admitted pts I just order a non-con chest CT. This way, the inpatient team will have a clear Dx and won’t overtreat or continue fishing. However, if the patient is going to be discharged, I perform an 8-zone lung ultrasound looking for signs of PNA. If any zones are positive, I discharge the patient with antibiotics. Now antibiotic selection in PNA is a big topic but I will say one thing. Azithro alone is NOT a sufficient treatment for CAP. Unfortunately, due to its overuse in the outpatient setting, macrolides are no longer effective against S. pneumoniae. Doxy is a reasonable outpatient choice if your regional antibiogram shows good coverage of S. pneumoniae, otherwise dual coverage with a B-lactam backbone is preferred. Lastly, stop ordering procalcitonin to diagnose PNA in the ED! The IDSA agrees… https://www.instagram.com/p/CQ4AX5QB15z/
Meme Star wars, you saved the clip My life in a nutshell. https://www.instagram.com/p/CQ8vYMNhCH5/
Clinical Cases Soft Tissue Abscess Abscess How to completely evaluate and abscess A 28 Y M presents with a painful area in his right gluteal fold. The area is red and indurated. You feel some fluctuance and think that this could represent an abscess. You perform the above POCUS prior to incision and drainage. What are the key aspects of evaluating an abscess that are missing from this assessment? -- Answer Below – Measure depth and size of the “collection” you are going to be cutting into, this way if you have already cut deep enough and aren’t getting any purulent drainage you can re-evaluate. Image in 2 planes to confirm that it is blind ending. If the collection connects to a vessel or runs off deep, then it probably is not something you should be trying to drain at the bedside. Put color flow on it. Color flow can help to show you that the structure you are looking not is not an abscess and instead is something with vascularity. The above images appear to have reactive “cobblestoning” surrounding a fairly homogenous structure. This finding represents cellulitis and can usually be found near an abscess. Now on ultrasound the contents of an abscess can range from anechoic (black) to something like this that is hard to distinguish from soft tissue. One maneuver that can help here is some light compression of the area. If this is truly an abscess you should see something called “pussastalsis.” Basically, the puss within the abscess should swirl around a bit with compression. There have been many a time when I have considered cutting into something, only to look with the ultrasound and find that the “abscess” in question was either a lymph node or something else I should not be slicing into. In this case despite a somewhat incomplete assessment, the team was able to easily perform and I&D on this abscess and prevent further discomfort or complications for the patient. This is just a reminder that even though something works, we should always aim for best practices when caring for our patients. https://www.instagram.com/p/CRT7pCnh5BV/
Yes Lecture Intern intro lecture Physics, cardiac, fast, lungs Lecture Series https://www.instagram.com/p/CRmh4YIjo-h/
Clinical Cases Gallbladder Cholecystitis Stone in neck sign, HIDA scan stone in neck (SIN) 43 Y female presents with acute onset RUQ pain after eating a hamburger. She appears uncomfortable. You decide to do a point of care ultrasound (POCUS) to see if you can make a bedside diagnosis to help expedite her care. You obtain the above images. What is your diagnosis next step in management? -- Answer Below – Diagnosis: Stone in neck, likely acute cholecystitis Management: Don’t forget gallstones alone does not mean cholecystitis! The first thing to note is that there are 2 stones in this image. The 2nd stone would be easy to miss if you weren’t paying attention. When doing gallbladder ultrasound any time you see posterior shadowing you should carefully investigate that area for stones causing the shadowing. Now the 2nd stone here is very near the neck of the gallbladder. Remember that an immobile stone in the neck of the gallbladder is a very specific sign for acute cholecystitis. Notice the image that is labeled roll, this is key to confirming a SIN sign. You must demonstrate that even after rolling the patient on their left side, the stone does not leave the GB neck. After demonstrating an immobile stone in the GB neck, you should reach out to your surgical colleges for assistance. While awaiting their consult, ED management should focus on symptomatic treatment and antibiotics. Even though acute cholecystitis is mostly thought to be an inflammatory process, secondary infection is possible and empiric antibiotics, while awaiting definitive surgical treatment, is still standard of care. Antibiotics should target gram negative rods as well as anaerobes and my normal regiment is Flagyl + Ceftriaxone (unless there are other complicating factors). Given that the only finding in this patient was a SIN sign, general surgery recommended a HIDA scan. The HIDA demonstrated findings consistent with acute cholecystitis and the patient was taken for cholecystectomy. There are 5 POCUS findings for acute cholecystitis. Do you remember them all? ...... 1. Thickened anterior GB wall >3mm 2. Pericholecystic fluid 3. Sonographic Murphy’s sign 4. Hydropic GB, which is a GB measuring >10cm long x 5cm wide 5. Non-mobile stone lodged in the GB neck (SIN sign) https://www.instagram.com/p/CRomncIhGok/
Yes The ED Resuscitation Part 1 ABC, IV, O2, Monitor initial evaluation of a critical patient The ED Resuscitation Learning to incorporate POCUS into critical patient care first requires you to learn to think like an ER doc. In EM we talk about 2 types of ED patients. The H&P patient and the ABCs patient. Today we are going to discuss the ABCs patient. The idea here is that in sick patients you should forgo a lengthy history and instead immediately start on your resuscitation. These cases can be stressful so there are a few mantras we always talk about to help with these critical patients. The first two are ABC and IV, O2, Monitor. Now in most departments these things will happen in tandem. IV, O2 and Monitor are usually taken care of by the nurses and techs, while as the doc you begin your Airway, Breathing, and Circulation assessment. However, it is important to remember that if you are at a shop with less staff or nurses who are less experienced in the ED you might need to give directions. Generally, when I walk into a room with a sick patient if things aren’t already moving, I start delegating tasks. In critical situations it is important to DELEGATE tasks. Make eye contact, point, or ask someone by name, POLITELY and CALMLY, to hook the patient up to the Monitor. Ask someone else to work on starting an IV. Especially when it comes to sick patients, keeping the energy in the room calm goes a LONG way. Like Scott Weingart say, “slow is smooth and smooth is fast.” Next, you need to assess the patient’s airway. The easiest way to do this is by asking the patient to speak. There are a lot of nuances regarding when to intubate a patient, but in general, if they can speak, you have some time. Once they are on the monitor if they are hypoxic put them on O2. Generally, I start with 5L by nasal cannula, but you may need to consider NRB or NIV. Once the first blood pressure is up on the monitor in non-trauma patients if the MAP is <65 I will just ask for a bag of LR to be hung while I am working on a plan. At this point you have addressed your initial ABCs, established an IV and have the patient on the monitor, from here you can start a brief H&P. Make sure to ask about allergies, medical hx, and a brief description of what got the patient into the current situation. https://www.instagram.com/p/CR1zUUjhj5V/
Featured Post type Major topic Diagnosis Series